RD Sharma Solutions for Class 12 Maths Chapter 2–Functions
RD Sharma Solutions for Class 12 Maths Chapter 2–Functions

Class 12: Maths Chapter 1 solutions. Complete Class 12 Maths Chapter 1 Notes.

Contents

RD Sharma Solutions for Class 12 Maths Chapter 2–Functions

RD Sharma 12th Maths Chapter 1, Class 12 Maths Chapter 1 solutions

Page No 2.31:

Question 1:

Give an example of a function
(i) which is one-one but not onto
(ii) which is not one-one but onto
(iii) which is neither one-one nor onto

ANSWER:

(i) which is one-one but not onto.

fZ → Z given by f(x)=3x+2

Injectivity:
Let x and y be any two elements in the domain (Z), such that f(x) = f(y).
 f(x)=f(y)
⇒3x + 2 =3y + 2
⇒3x = 3y
x = y
f(x) = f(y) ⇒x = y
So, f is one-one.

Surjectivity:
Let y be any element in the co-domain (Z), such that f(x) = y for some element x in Z (domain).
f(x) = y
⇒3x + 2 = y
⇒3x = y – 2

x = y-23. It may not be in the domain (Z) because if we take y = 3,x = y-23 = 3-23 = 13∉ domain Z.
So, for every element in the co domain there need not be any element in the domain such that f(x) = y.
Thus, f is not onto.

(ii) which is not one-one but onto.
fZ → N ∪ {0} given by f(x) = |x|

Injectivity:
Let and y be any two elements in the domain (Z), such that f(x) = f(y).
⇒|x| = |y|
x= ± y
So, different elements of domain f may give the same image.
So, f is not one-one.

Surjectivity:
Let y be any element in the co domain (Z), such that f(x) = y for some element x in Z (domain).
f(x) = y
⇒|x| = y
x = ± y, which is an element in Z (domain).
So, for every element in the co-domain, there exists a pre-image in the domain.
Thus, f is onto.

(iii) which is neither one-one nor onto.

fZ → Z given by f(x) = 2x2 + 1

Injectivity:
Let x and y be any two elements in the domain (Z), such that f(x) = f(y).
f(x) = f(y)⇒2x2+1 = 2y2+1⇒2x2 = 2y2⇒x2 = y2⇒x = ±y
So, different elements of domain f may give the same image.
Thus, f is not one-one.

Surjectivity:
Let y be any element in the co-domain (Z), such that f(x) = y for some element x in Z (domain).
f(x) = y
⇒2x2+1=y⇒2x2=y-1⇒x2=y-12⇒x=±√y-12, ∉ Z always.For example, if we take, y = 4,x=±√y-12=±√4-12=±√32, ∉ ZSo, x may not be in Z (domain).

Thus, f is not onto.

Page No 2.31:

Question 2:

Which of the following functions from A to B are one-one and onto?
(i) f1 = {(1, 3), (2, 5), (3, 7)} ; A = {1, 2, 3}, B = {3, 5, 7}
(ii) f2 = {(2, a), (3, b), (4, c)} ; A = {2, 3, 4}, B = {abc}
(iii) f3 = {(ax), (bx), (cz), (dz)} ; A = {abcd,}, B = {xyz}

ANSWER:

(i) f1 = {(1, 3), (2, 5), (3, 7)} ; A = {1, 2, 3}, B = {3, 5, 7}

Injectivity:
f1 (1) = 3
f1 (2) = 5
f1 (3) = 7
⇒Every element of has different images in B.
So, f1 is one-one.

Surjectivity:
Co-domain of f1 = {3, 5, 7}
Range of f1 =set of images  =  {3, 5, 7}
⇒Co-domain = range
So, f1 is onto.

(ii) f2 = {(2, a), (3, b), (4, c)} ; A = {2, 3, 4}, B = {abc}

Injectivity:
f2 (2) = a
f2 (3) = b
f2 (4) = c
⇒Every element of has different images in B.
So, f2 is one-one.

Surjectivity:
Co-domain of f2 = {abc}
Range of f2 = set of images = {abc}
⇒Co-domain = range
So, f2 is onto.

(iii) f3 = {(ax), (bx), (cz), (dz)} ; A = {abcd,}, B = {xyz}

Injectivity:
f3 (a) = x
f3 (b) = x
f3 (c) = z
f3 (d) = z

a and b have the same image x. (Also c and d have the same image z)
So, f3 is not one-one.

Surjectivity:
Co-domain of f1 ={x, y, z}
Range of f1 =set of images  =  {x, z}
So, the co-domain  is not same as the range.
So, f3 is not onto.

Page No 2.31:

Question 3:

Prove that the function f : N → N, defined by f(x) = x2 + x + 1, is one-one but not onto.

ANSWER:

f : N → N, defined by f(x) = x2 + x + 1

Injectivity:
Let x and y be any two elements in the domain (N), such that f(x) = f(y).
⇒x2+x+1=y2+y+1⇒(x2-y2)+(x-y)=0⇒(x+y)(x-y)+(x-y)=0⇒(x-y)(x+y+1)=0⇒x-y=0     [ ( x+y+1) cannot be zero because x and y are natural numbers]⇒x=y
 So, is one-one.

Surjectivity:
The minimum number in N is 1.When x=1,x2+x+1=1+1+1=3⇒x2+x+1≥3, for every x in N.⇒f(x) will not assume the values 1 and 2.So, f is not onto.

Page No 2.31:

Question 4:

Let A = {−1, 0, 1} and f = {(xx2) : x ∈ A}. Show that f : A → A is neither one-one nor onto.

ANSWER:

A = {−1, 0, 1} and f = {(xx2) : x ∈ A}
Given, f(x) = x2

Injectivity:
f(1) = 12=1 and
f(-1)=(-1)2=1

⇒1 and -1 have the same images.
So, f  is not one-one.

Surjectivity:
Co-domain of  f  = {-1, 0, 1}

f(1) = 12 = 1,
f(-1) = (-1)2 = 1 and
f(0) = 0
⇒Range of f  = {0, 1}
So, both are not same.
Hence, f  is not onto.

Page No 2.31:

Question 5:

Classify the following functions as injection, surjection or bijection :
(i) f : N → N given by f(x) = x2
(ii) f : Z → Z given by f(x) = x2
(iii) f : N → N given by f(x) = x3
(iv) f : Z → Z given by f(x) = x3
(v) f : R → R, defined by f(x) = |x|
(vi) f : Z → Z, defined by f(x) = x2 + x
(vii) f : Z → Z, defined by f(x) = x − 5
(viii) f : R → R, defined by f(x) = sinx
(ix) f : R → R, defined by f(x) = x3 + 1
(x) f : R → R, defined by f(x) = x3 − x
(xi) f : R → R, defined by f(x) = sin2x + cos2x
(xii) f : Q − {3} → Q, defined by f(x)=2x+3x-3
(xiii) f : Q → Q, defined by f(x) = x3 + 1
(xiv) f : R → R, defined by f(x) = 5x3 + 4
(xv) f : R → R, defined by f(x) = 3 − 4x
(xvi) f : R → R, defined by f(x) = 1 + x2
(xvii) f : R → R, defined by f(x) = xx2+1                                                                                                                    [NCERT EXEMPLAR]

ANSWER:

(i) f : N → N, given by f(x) = x2

Injection test:

Let x and y be any two elements in the domain (N), such that f(x) = f(y).

f(x)=f(y)

x2=y2x=y       (We do not get ± because x and y are in N)

So, f is an injection .

Surjection test:

Let y be any element in the co-domain (N), such that f(x) = y for some element x in N (domain).

f(x) = y

x2=yx=√y, which may not be in N.For example, if y=3,x=√3 is not in N.

So, is not a surjection.

So, f is not a bijection.

(ii) f : Z → Z, given by f(x) = x2

Injection test:

Let x and y be any two elements in the domain (Z), such that f(x) = f(y).

f(x= f(y)

x2=y2xy

So, f is not an injection .

Surjection test:

Let y be any element in the co-domain (Z), such that f(x) = y for some element x in Z (domain).

f(x) = y

x2=yx=±√y which may not be in Z.For example, if y=3,x=±√3 is not in Z.

So, is not a surjection.

So, f is not a bijection.

(iii) f : N → N, given by f(x) = x3

Injection test:

Let x and y be any two elements in the domain (N), such that f(x) = f(y).

f(x= f(y)

x3=y3x=y

So, f is an injection .

Surjection test:

Let y be any element in the co-domain (N), such that f(x) = y for some element x in (domain).

f(x) = y

x3=yx=3√ywhich may not be in N.For example, if y=3,x=3√3 is not in N.

So, is not a surjection and  f is not a bijection.

(iv) f : Z → Z, given by f(x) = x3

Injection test:

Let x and y be any two elements in the domain (Z), such that f(x) = f(y)

f(x= f(y)

x3=y3x=y

So, f is an injection.

Surjection test:

Let y be any element in the co-domain (Z), such that f(x) = y for some element x in Z (domain).

f(x) = y

x3=yx=3√y which may not be in Z.For example, if y=3,x=3√3 is not in Z.

So, is not a surjection and f is not a bijection.

(v) f : R → R, defined by f(x) = |x|

Injection test:

Let x and y be any two elements in the domain (R), such that f(x) = f(y)

f(x= f(y)

|x|=|y|xy

So, f is not an injection .

Surjection test:

Let y be any element in the co-domain (R), such that f(x) = y for some element x in R (domain).

f(x) = y

|x|=yxyZ

So, is a surjection and  f is not a bijection.

(vi) f : Z → Z, defined by f(x) = x2 + x

Injection test:

Let x and y be any two elements in the domain (Z), such that f(x) = f(y).

f(x= f(y)

x2+x=y2+yHere, we cannot say that x = y.For example, x = 2 and y = – 3Then, x2+x=22+2= 6y2+y=(-3)2-3= 6So, we have two numbers 2 and -3 in the domain Z whose image is same as 6.

So, f is not an injection .

Surjection test:

Let y be any element in the co-domain (Z), such that f(x) = y for some element x in Z (domain).

f(x) = y

x2+x=yHere, we cannot say xZ.For example, y =-4.x2+x=-4x2+x+4=0x=-1±√-152=-1±i√152 which is not in Z.

So, is not a surjection and  f is not a bijection.

(vii) f : Z → Z, defined by f(x) = x − 5

Injection test:

Let x and y be any two elements in the domain (Z), such that f(x) = f(y).

f(x= f(y)

x – 5 = y – 5

x = y

So, f is an injection .

Surjection test:

Let y be any element in the co-domain (Z), such that f(x) = y for some element x in Z (domain).

f(x) = y

– 5 = y

x = y + 5, which is in Z.

So, is a surjection and f is a bijection.

(viii) f : R → R, defined by f(x) = sinx

Injection test:

Let x and y be any two elements in the domain (R), such that f(x) = f(y).

f(x= f(y)

sinx=sinyHere, x may not be equal to y because sin0=sinπ.So, 0 and π have the same image 0.

So, f is not an injection .

Surjection test:

Range of f = [-1, 1]

Co-domain of R

Both are not same.

So, f is not a surjection and f is not a bijection.

(ix) f : R → R, defined by f(x) = x3 + 1

Injection test:

Let x and y be any two elements in the domain (R), such that f(x) = f(y).

f(x= f(y)

x3+1=y3+1x3=y3x=y

So, f is an injection.

Surjection test:

Let y be any element in the co-domain (R)such that f(x) = y for some element x in (domain).

f(x) = y

x3+1=yx=3√y-1∈R

So, is a surjection.

So, f is a bijection.

(x) f : R → R, defined by f(x) = x3 − x

Injection test:

Let x and y be any two elements in the domain (R), such that f(x) = f(y).

f(x= f(y)

x3-x=y3-yHere, we cannot say x=y.For example, x=1 and y=-1x3-x=1-1=0y3-y=(-1)3-(-1)-1+1=0So, 1 and -1 have the same image 0.

So, f is not an injection.

Surjection test:

Let y be any element in the co-domain (R), such that f(x) = y for some element x in (domain).

f(x) = y

x3-x=yBy observation we can say that there exist some x in R, such that x3-x=y.

So, is a surjection and  f is not a bijection.

(xi) f : R → R, defined by f(x) = sin2x + cos2x

f(x) = sin2x + cos2= 1

So, f(x) = 1 for every x in R.

So, for all elements in the domain, the image is 1.

So, f is not an injection.

Range of = {1}

Co-domain of f = R

Both are not same.

So, f is not a surjection and  is not a bijection.

(xii) f : Q − {3} → Q, defined by f(x)=2x+3x-3

Injection test:

Let x and y be any two elements in the domain (Q − {3}), such that f(x) = f(y).

f(x) = f(y)

2x+3x-3=2y+3y-3(2x+3)(y-3)=(2y+3)(x-3)2xy-6x+3y-9=2xy-6y+3x-99x=9yx=y

So, f is an injection.

Surjection test:

Let y be any element in the co-domain (Q − {3}), such that f(x) = y for some element x in Q (domain).

f(x) = y

2x+3x-3=y2x+3=xy-3y2xxy=-3y-3x(2-y)=-3(y+1)x=3(y+1)y-2, which is not defined at y=2.

So, f is not a surjection and f is not a bijection.

(xiii) f : Q → Q, defined by f(x) = x3 + 1

Injection test:

Let x and y be any two elements in the domain (Q), such that f(x) = f(y).

f(x= f(y)

x3+1=y3+1x3=y3x=y

So, f is an injection .

Surjection test:

Let y be any element in the co-domain (Q), such that f(x) = y for some element x in Q (domain).

f(x) = y

x3+1=yx=3√y-1, which may not be in Q.For example, if y= 8,x3+1= 8x3=7x=3√7, which is not in Q.

So, f is not a surjection and f is not a bijection.

So, is a surjection and f is a bijection.

(xiv) f : R → R, defined by f(x) = 5x3 + 4

Injection test:

Let x and y be any two elements in the domain (R), such that f(x) = f(y).

f(x= f(y)

5x3+4 = 5y3+45x3= 5y3x3= y3x=y

So, f is an injection .

Surjection test:

Let y be any element in the co-domain (R), such that f(x) = y for some element x in (domain).

f(x) = y

5x3+4=y5x3=y-4x3=y-45x=3√y-45∈R

So, is a surjection and f is a bijection.

(xv) f : R → R, defined by f(x) = 3 − 4x

Injection test:

Let x and y be any two elements in the domain (R), such that f(x) = f(y).

f(x= f(y)

3-4x=3-4y-4x=-4yxy

So, f is an injection .

Surjection test:

Let y be any element in the co-domain (R), such that f(x) = y for some element x in (domain).

f(x) = y

3-4x=y4x=3-yx=3-y4∈R

So, is a surjection and f is a bijection.

(xvi) f : R → R, defined by f(x) = 1 + x2

Injection test:

Let x and y be any two elements in the domain (R), such that f(x) = f(y).

f(x= f(y)

1+x2=1+y2x2=y2x= ±y

So, f is not an injection.

Surjection test:

Let y be any element in the co-domain (R), such that f(x) = y for some element x in (domain).

f(x) = y

1+x2=yx2=y-1x=±√y-1 which may not be in RFor example, if y=0,x=±√-1=±i is not in R.

So, is not a surjection and f is not a bijection.

(xvii)  f : R → R, defined by f(x) = xx2+1

Injection test:

Let x and y be any two elements in the domain (R), such that f(x) = f(y).

f(x= f(y)

xx2+1=yy2+1xy2+x=x2y+yxy2-x2y+xy=0-xy(-y+x)+1(xy)=0(xy)(1-xy)=0x=y or x=1y

So, f is not an injection.

Surjection test:

Let y be any element in the co-domain (R), such that f(x) = y for some element x in (domain).

f(x) = y

xx2+1=yyx2-x+y=0x=-(-1)±√1-4y22y, if y≠0=1±√1-4y22y, which may not be in RFor example, if y=1, thenx=1±√1-42=1±i√32, which is not in RSo, f is not surjection and f is not bijection.

So, is not a surjection and f is not a bijection.

Page No 2.31:

Question 6:

If f : A → B is an injection, such that range of f = {a}, determine the number of elements in A.

ANSWER:

Range of f = {a}
So, the number of images of  = 1
Since, f  is an injection, there will be exactly one image for each element of f .
So, number of elements in A = 1.

Page No 2.31:

Question 7:

Show that the function f : R − {3} → R − {2} given by f(x)=x-2x-3 is a bijection.

ANSWER:

f : R − {3} → R − {2} given by
f(x)=x-2x-3
Injectivity:
Let x and y be any two elements in the domain (R − {3}), such that f(x) = f(y).
 f(x) = f(y)
⇒x-2x-3=y-2y-3⇒(x-2)(y-3)=(y-2)(x-3)⇒xy-3x-2y+6=xy-3y-2x+6⇒x=y

So, f is one-one.

Surjectivity:
Let y be any element in the co-domain (R − {2}), such that f(x) = y for some element x in R − {3} (domain).
f(x) = y
x-2x-3=yx-2=xy-3yxyx=3y-2⇒x(y-1)=3y-2⇒x=3y-2y-1, which is in R-{3}
So, for every element in the co-domain, there exists some pre-image in the domain.
f  is onto.
Since, f  is both one-one and onto, it is a bijection.

Page No 2.32:

Question 8:

Let A = [-1, 1]. Then, discuss whether the following functions from A to itself are one-one, onto or bijective:

(i) f(x) = x2                                (ii) g(x) = |x|                                (iii) h(x) = x2                                                               [NCERT EXEMPLAR]

ANSWER:

(i) f : A → A, given by f(x) = x2

Injection test:

Let x and y be any two elements in the domain (A), such that f(x) = f(y).

f(x) = f(y)

x2 = y2

x = y

So, f is one-one.

Surjection test:

Let y be any element in the co-domain (A), such that f(x) = y for some element x in A (domain)

f(x) = y

x2 = y

x = 2y, which may not be in A.

For example, if y = 1, then

x = 2, which is not in A.

So, f is not onto.

So, f is not bijective.

(ii) g(x) = |x|

Injection test:

Let x and y be any two elements in the domain (A), such that f(x) = f(y).

f(x) = f(y)

|x| = |y|

x = ±y

So, f is not one-one.

Surjection test:

For y = -1, there is no value of x in A.

So, f is not onto.

So, f is not bijective.

(iii) h(x) = x2

Injection test:

Let x and y be any two elements in the domain (A), such that f(x) = f(y).

f(x) = f(y)

x2 = y2

x = ±y

So, f is not one-one.

Surjection test:

For y = -1, there is no value of x in A.

So, f is not onto.

So, f is not bijective.

 

Page No 2.32:

Question 9:

Are the following set of ordered pairs functions? If so, examine whether the mapping is injective or surjective:

(i) {(xy) : x is a person, y is the mother of x}
(ii) {(ab) : a is a person, b is an ancestor of a}                                                                                                               [NCERT EXEMPLAR]

ANSWER:

(i) f = {(xy) : x is a person, y is the mother of x}

As, for each element x in domain set, there is a unique related element y in co-domain set.

So, f is the function.

Injection test:

As, y can be mother of two or more persons

So, f is not injective.

Surjection test:

For every mother y defined by (xy), there exists a person x for whom y is mother.

So, f is surjective.

Therefore, f is surjective function.

(ii) g = {(ab) : a is a person, b is an ancestor of a}

Since, the ordered map (ab) does not map ‘a‘ – a person to a living person.
So, g is not a function.

Page No 2.32:

Question 10:

Let A = {1, 2, 3}. Write all one-one from A to itself.

ANSWER:

A ={1, 2, 3}
Number of elements in  = 3
Number of one-one functions = number of ways of arranging 3 elements = 3! = 6

(i) {(1, 1), (2, 2), (3, 3)}
(ii) {(1, 1), (2, 3), (3, 2)}
(iii) {(1, 2 ), (2, 2), (3, 3 )}
(iv) {(1, 2), (2, 1), (3, 3)}
(v) {(1, 3), (2, 2), (3, 1)}
(vi) {(1, 3), (2, 1), (3,2 )}

Page No 2.32:

Question 11:

If f : R → R be the function defined by f(x) = 4x3 + 7, show that f is a bijection.

ANSWER:

Injectivity:
Let x and y be any two elements in the domain (R), such that f(x) = f(y)
⇒4x3+7=4y3+7⇒4x3=4y3⇒x3=y3⇒x=y
So, is one-one.

Surjectivity:
Let y be any element in the co-domain (R), such that f(x) = y for some element x in (domain).
f(x) = y
⇒4x3+7=y⇒4x3=y-7⇒x3=y-74⇒x=3√y-74∈R
So, for every element in the co-domain, there exists some pre-image in the domain.
is onto.
Since, f is both one-to-one and onto, it is a bijection.

Page No 2.32:

Question 12:

Show that the exponential function f : R → R, given by f(x) = ex, is one-one but not onto. What happens if the co-domain is replaced by R+0 (set of all positive real numbers)?

ANSWER:

f : R → R, given by f(x) = ex

Injectivity:
Let x and y be any two elements in the domain (R), such that f(x) = f(y)
 f(x)=f(y)
⇒ex=ey⇒x=y

So, is one-one.

Surjectivity:
We know that range of ex is (0, ∞) = R+
Co-domain = R
Both are not same.
So, f is not onto.

If the co-domain is replaced by R+, then the co-domain and range become the same and in that case, f is onto and hence, it is a bijection.

Page No 2.32:

Question 13:

Show that the logarithmic function f : R+0→R given by f(x)=loga xa>0 is a bijection.

ANSWER:

f:R+R given by f(x)= loga xa>0
Injectivity:
Let x and y be any two elements in the domain (N), such that f(x= f(y).
 f(x) = f(y)
loga x=loga yx=y
So, f is one-one.

Surjectivity:
Let y be any element in the co-domain (R), such that f(x) = y for some element x in R+ (domain).
f(x) = y
loga x=yx=ay ∈R+
So, for every element in the co-domain, there exists some pre-image in the domain.
f is onto.
Since f is one-one and onto, it is a bijection.

Page No 2.32:

Question 14:

If A = {1, 2, 3}, show that a one-one function f : A → A must be onto.

ANSWER:

A ={1, 2, 3}
Number of elements in  = 3
Number of one – one functions = number of ways of arranging 3 elements = 3! = 6
So, the possible one -one functions can be the following:

(i) {(1, 1), (2, 2), (3, 3)}
(ii) {(1, 1), (2, 3), (3, 2)}
(iii) {(1, 2 ), (2, 2), (3, 3 )}
(iv) {(1, 2), (2, 1), (3, 3)}
(v) {(1, 3), (2, 2), (3, 1)}
(vi) {(1, 3), (2, 1), (3,2 )}
Here, in each function, range = {1, 2, 3}, which is same as the co-domain.
So, all the functions are onto.

Page No 2.32:

Question 15:

If A = {1, 2, 3}, show that a onto function f : A → A must be one-one.

ANSWER:

A ={1, 2, 3}
Possible onto functions from A to A can be the following:

(i) {(1, 1), (2, 2), (3, 3)}
(ii) {(1, 1), (2, 3), (3, 2)}
(iii) {(1, 2 ), (2, 2), (3, 3 )}
(iv) {(1, 2), (2, 1), (3, 3)}
(v) {(1, 3), (2, 2), (3, 1)}
(vi) {(1, 3), (2, 1), (3,2 )}

Here, in each function, different elements of the domain have different images.
So, all the functions are one-one.

Page No 2.32:

Question 16:

Find the number of all onto functions from the set A = {1, 2, 3, …, n} to itself.

ANSWER:

We know that every onto function from A to itself is one-one.
So, the number of one-one functions = number of bijections = n!

Page No 2.32:

Question 17:

Give examples of two one-one functions f1 and f2 from R to R, such that f1 + f2 : R → R. defined by (f1 + f2) (x) = f1 (x) + f2 (x) is not one-one.

ANSWER:

We know that f1R → R, given by f1(x)=x, and f2(x)=-x are one-one.
Proving f1 is one-one:
Let f1(x)=f1(y)⇒x=y
So, fis one-one.

Proving f2 is one-one:
Let f2(x)=f2(y)⇒-x=-yx=y
So, f2  is one-one.

Proving (f1 + f2) is not one-one:
Given:
(f1 + f2) (x) = f1 (x) + f2 (x)= x + (-x) =0
So, for every real number x, (f1 + f2) (x)=0
So, the image of ever number in the domain is same as 0.
Thus, (f1 + f2) is not one-one.

Page No 2.32:

Question 18:

Give examples of two surjective functions f1 and f2 from Z to Z such that f1 + f2 is not surjective.

ANSWER:

We know that f1R → R, given by f1(x) = x, and f2(x) = -x are surjective functions.
Proving f1 is surjective :
Let y be an element in the co-domain (R), such that f1(x) = y.
f1(x) = y
⇒x = y, which is in R.
Sofor every element in the co-domain, there exists some pre-image in the domain.1(x)=f1(y)x=y
So, f1is surjective .

Proving f2 is surjective :Let f2(x)=f2(y)−x=−yx=y
Let y be an element in the co domain (R) such that f2(x) = y.
 f2(x) = y
⇒x = y, which is in R.
Sofor every element in the co-domain, there exists some pre-image in the domain.1(x)=f1(y)x=y
So, f2 is surjective .

Proving (f1 + f2) is not surjective :
Given:
(f1 + f2) (x) = f1 (x) + f2 (x)= x + (-x) =0
So, for every real number x, (f1 + f2) (x)=0
Sothe image of every number in the domain is same as 0.
Range = {0}
Co-domain = R
So, both are not same.
So, f1 + f2 is not surjective.

Page No 2.32:

Question 19:

Show that if f1 and f2 are one-one maps from R to R, then the product ff2 : RR defined by (ff2) (x)=f1 (xf2 (x) need not be one-one.

ANSWER:

We know that f1R → R, given by f1(x) = x, and f2(x) = x are one-one.
Proving f1 is one-one:
Let x and y be two elements in the domain R, such that
f1(x) = f1(y)
⇒x = y
et f1(x)=f1(y)x=y
So, f1 is one-one.

Proving f2 is one-one:
Let x and y be two elements in the domain R, such that
f2(x) = f2(y)
⇒x = y
et f1(x)=f1(y)x=y
So, f2 is one-one.

Proving f1 × f2 is not one-one:
Given:
(f1 × f2)(x)=f1 (x) × f2 (x)=x × x=x2Let x and y be two elements in the domain R, such that(f1 × f2)(x)=(f1 × f2)(y)⇒x2 = y2⇒x=± ySo, (f1 × f2) is not one-one.ff2

Page No 2.32:

Question 20:

Suppose f1 and f2 are non-zero one-one functions from R to R. Is f1f2 necessarily one-one? Justify your answer. Here, f1f2:RR is given by (f1f2) (x)=f1 (x)f2 (x) for all xR.

ANSWER:

We know that f1R → R, given by f1(x)=x3 and f2(x)=x are one-one.
Injectivity of f1:
Let x and y be two elements in the domain R, such that
f1(x)=f2(y)⇒x3=yx=3√yRLet f1(x)=f1(y)x=y
So, f1 is one-one.

Injectivity of f2:
Let x and y be two elements in the domain R, such that
f2(x)=f2(y)⇒x=y ⇒xR.Let f2(x)=f2(y)−x=−yx=y
So, f2  is one-one.

Proving f1f2is not one-one:
Given that f1f2(x)=f1(x)f2(x)=x3x=x2
Let x and y be two elements in the domain R, such that

f1f2(x)=f1f2(y)⇒x2=y2⇒xy
So, f1f2 is not one-one.

Page No 2.32:

Question 21:

Given A = {2, 3, 4}, B = {2, 5, 6, 7}. Construct an example of each of the following:

(i) an injective map from A to B
(ii) a mapping from A to B which is not injective
(iii) a mapping from A to B.

ANSWER:

(i) {(2, 7), (3, 6), (4, 5)}

(ii) {(2, 2), (3, 2), (4, 5)}

(iii) {(2, 5), (3, 6), (4, 7)}

Disclaimer: There are many more possibilities of each case.

Page No 2.32:

Question 22:

Show that f : R → R, given by f(x) = x – [x], is neither one-one nor onto.

ANSWER:

We have, f(x) = x – [x]

Injection test:

f(x) = 0 for all x ∈ Z

So, f is a many-one function.

Surjection test:

Range (f) = [0, 1) ≠ R.

So, f is an into function.

Therefore, f is neither one-one nor onto.

Page No 2.32:

Question 23:

Let f : N → N be defined by

f(n)={n+1, if n is oddn-1, if n is even

Show that f is a bijection.                                                                                                                                                   [CBSE 2012, NCERT]

ANSWER:

We have,f(n)={n+1, if n is oddn-1, if n is evenInjection test:Case I: If n is odd,Let xyN such that f(x)=f(y)As, f(x)=f(y)⇒x+1=y+1⇒x=yCase II: If n is even,Let xyN such that f(x)=f(y)As, f(x)=f(y)⇒x-1=y-1⇒x=ySo, f is injective.Surjection test:Case I: If n is odd,As, for every nN, there exists y=n-1 in N such thatf(y)=f(n-1)=n-1+1=nCase II: If n is even,As, for every nN, there exists y=n+1 in N such thatf(y)=f(n+1)=n+1-1=nSo, f is surjective.So, f is a bijection.

Page No 2.46:

Question 1:

Find gof and fog when f : R → R and g : R → R are defined by
(i) f(x) = 2x + 3                and       g(x) = x2 + 5
(ii) f(x) = 2x + x2             and       g(x) = x3
(iii) f(x) = x2 + 8              and       g(x) = 3x3 + 1
(iv) f(x) = x                       and       g(x) = |x|
(v) f(x) = x2 + 2x − 3       and       g(x) = 3x − 4
(vi) f(x) = 8x3                   and       g(x) = x1/3

ANSWER:

Given, f : R → R and g : R → R
So, gof R → R  and fog : R → R

(i) f(x) = 2x + 3  and g(x) = x2 + 5
Now, (gof) (x)
= g (f (x))
= g 
(2x +3)
(2x + 3)2 + 5
4x2+ 9 + 12x +5
=4x2+  12x + 14

(fog) (x)
=f (g (x))
= f (x2 
+ 5)
= 2 (x2 + 5) +3
2 x2+ 10 + 3
= 2x2 + 13

(ii) f(x) = 2x + x2 and g(x) = x3
(gof) (x)=g (f (x))=g (2x+x2)=(2x+x2)3(fog) (x)=f (g (x))=f (x3)=2 (x3)+(x3)2=2x3+x6

(iii) f(x) = x2 + 8  and g(x) = 3x3 + 1
(gof) (x)=g (f(x))=g (x2+8)=3 (x2+8)3+1(fog) (x)=f (g (x))=f (3x3+1)=(3x3+1)2+8=9x6+6x3+1+8=9x6+6x3+9

(iv) f(x) = x and g(x) = |x|
(gof) (x)=g (f(x))=g (x)=|x|(fog) (x)=f (g (x))=f (|x|)=|x|

(v) f(x) = x2 + 2x − 3 and g(x) = 3x − 4
(gof) (x)=g (f(x))=g (x2+2x-3)=3 (x2+2x-3)-4=3x2+6x-9-4=3x2+6x-13(fog) (x)=f (g (x))=f (3x-4)=(3x-4)2+2 (3x-4)-3=9x2+16-24x+6x-8-3=9x2-18x+5

(vi) f(x) = 8x3 and g(x) = x1/3
(gof) (x)=g (f (x))=g (8x3)=(8x3)13=[(2x)3]13=2x(fog) (x)=f (g (x))=f (x13)=8 (x13)3=8x

Page No 2.46:

Question 2:

Let f = {(3, 1), (9, 3), (12, 4)} and g = {(1, 3), (3, 3) (4, 9) (5, 9)}. Show that gof and fog are both defined. Also, find fog and gof.

ANSWER:

f = {(3, 1), (9, 3), (12, 4)} and g = {(1, 3), (3, 3) (4, 9) (5, 9)}

: {3, 9, 12} → {1, 3,4} and : {1, 3, 4, 5} → {3, 9}

Co-domain of f is a subset of the domain of g.
So, gof exists and gof : {3, 9, 12} → {3, 9}
(gof) (3)=g (f (3))=g (1)=3(gof) (9)=g (f (9))=g (3)=3(gof) (12)=g (f (12))=g (4)=9⇒gof ={(3, 3), (9, 3), (12, 9)}
Co-domain of g is a subset of the domain of f.
So, fog exists and fog : {1, 3, 4, 5} → {3, 9, 12}
(fog) (1)=f (g (1))=f (3)=1(fog) (3)=f (g (3))=f (3)=1(fog) (4)=f (g (4))=f (9)=3(fog) (5)=f (g (5))=f (9)=3⇒fog={(1, 1), (3, 1), (4, 3), (5, 3)}

Page No 2.46:

Question 3:

Let f = {(1, −1), (4, −2), (9, −3), (16, 4)} and g = {(−1, −2), (−2, −4), (−3, −6), (4, 8)}. Show that gof is defined while fog is not defined. Also, find gof.

ANSWER:

f = {(1, −1), (4, −2), (9, −3), (16, 4)} and g = {(−1, −2), (−2, −4), (−3, −6), (4, 8)}
: {1, 4, 9, 16} → {-1, -2, -3, 4} and g : {-1, -2, -3, 4} → {-2, -4, -6, 8}
Co-domain of f = domain of g
So, gof exists and gof : {1, 4, 9, 16} → {-2, -4, -6, 8}
(gof) (1)=g (f (1))=g (-1)=-2(gof) (4)=g (f (4))=g (-2)=-4(gof) (9)=g (f (9))=g (-3)=-6(gof) (16)=g (f (16))=g (4)=8So, gof={(1, -2), (4, -4), (9, -6), (16, 8)}

But the co-domain of g is not same as the domain of f.
So, fog does not exist.

Page No 2.46:

Question 4:

Let A = {abc}, B = {u vw} and let f and g be two functions from A to B and from B to A, respectively, defined as :

    f = {(av), (bu), (cw)}, g = {(ub), (va), (wc)}.

Show that f and g both are bijections and find fog and gof.

ANSWER:

Proving f is a bijection:
f = {(av), (bu), (cw)} and : A → B
Injectivity of f: No two elements of have the same image in B.
So, f is one-one.
Surjectivity of f: Co-domain of f = {u vw}
Range of = {u vw}
Both are same.
So,  f is onto.
Hence, f is a bijection.

Proving is a bijection:
g = {(ub), (va), (wc)} and B → A
Injectivity of g: No two elements of B  have the same image in A.
So, g is one-one.
Surjectivity of g: Co-domain of g = {abc}
Range of g = {abc}
Both are the same.
So, g is onto.
Hence, g is a bijection.

Finding  fog:
Co-domain of g is same as the domain of f.
So, fog exists and fog : {u vw→ {u vw}
(fog) (u)=f (g (u))=f (b)=u(fog) (v)=f (g (v))=f (a)=v(fog) (w)=f (g (w))=f (c)=wSo, fog ={(uu), (vv), (ww)}

Finding gof:
Co-domain of f is same as the domain of g.
So, fog exists and gof : {abc→ {abc}
(gof) (a)=g (f (a))=g (v)=a(gof) (b)=g (f (b))=g (u)=b(gof) (c)=g (f (c))=g (w)=cSo, gof={(aa), (bb), (cc)}


Page No 2.46:

Question 5:

Find  fog (2) and gof (1) when : f : R → R ; f(x) = x2 + 8 and g : R → Rg(x) = 3x3 + 1.

ANSWER:

(fog) (2)=f (g (2))=f(3×23+1)=f(25)=252+8=633(gof) (1)=g (f (1))=g (12+8)=g (9)=3×93+1=2188

Page No 2.46:

Question 6:

Let R+ be the set of all non-negative real numbers. If f : R+ → R+ and g : R+ → R+ are defined as f(x)=x2 and g(x)=+√x, find fog and gof. Are they equal functions?

ANSWER:

Given,  f : R+ → R+ and g : R+ → R+
So,  fog R+ → R+  and gof R+ → R+
Domains of fog  and gof  are the same.
(fog) (x)=f (g (x))=f (√x)=(√x)2=x(gof) (x)=g (f (x))=g (x2)=√x2=xSo, (fog) (x)=(gof) (x),∀xR+
Hence, fog gof

Page No 2.46:

Question 7:

Let f : R → R and g : R → R be defined by f(x) = x2 and g(x) = x + 1. Show that fog ≠ gof.

ANSWER:

Given,  f : R → R and g : R → R.
So, the domains of f and g are the same.

(fog) (x)=f (g (x))=f (x+1)=(x+1)2=x2+1+2x(gof) (x)=g (f (x))=g (x2)=x2+1
So,  fog ≠ gof

Page No 2.46:

Question 8:

Let f : R → R and g : R → R be defined by f(x) = + 1 and g(x) = x − 1. Show that fog = gof = IR.

ANSWER:

Given,  f : R → R and g : R → R
fog :  R → R and gof R → R (Also, we know that IR : R → R)
So, the domains of all fog, gof and IR are the same.
(fog) (x)=f (g (x))=f (x-1)=x-1+1=x=IR (x)      … (1)(gof) (x)=g (f (x))=g (x+1)=x+1-1=x=IR (x)      … (2)From (1) and (2), (fog) (x)=(gof) (x)=IR (x), ∀xRHence, fog=gof=IR

Page No 2.46:

Question 9:

Verify associativity for the following three mappings : f : N → Z0 (the set of non-zero integers), g : Z0 → Q and h : Q → R given by f(x) = 2xg(x) = 1/x and h(x) = ex.

ANSWER:

Given that f : N → Z0 , g : Z0 → Q and h : Q → R .
gof → Q  and hog Z0 → R
h o (gof ) → and (hogo f→ R
So, both have the same domains.
(gof) (x)=g (f (x))=g (2x)=12x         …(1)(hog) (x)=h (g (x))=h (1x)=e1x       …(2)Now,(h o(gof)) (x)=h((gof) (x))=h (12x)=e12x           [from (1)]((hogo f)(x)=(hog) (f (x))= (hog) (2x)=e12x    [from (2)]⇒(h o(gof)) (x)=((hogo f)(x), ∀xNSo, h o(gof)=(hogo f
Hence, the associative property has been verified.

Page No 2.46:

Question 10:

Consider f : N → Ng : N → N and h : N → R defined as f(x) = 2xg(y) = 3y + 4 and h(z) = sin z for all xyz ∈ N. Show that ho (gof) = (hogof.

ANSWER:

Given, f : N → Ng : N → N and h : N → R
gof N → and hog N → R
ho (gof) : N → R and (hogof N → R
So, both have the same domains.
(gof) (x)=g (f (x))=g (2x)=3 (2x)+4=6x+4  …(1)(hog) (x)=h(g (x))=h (3x+4)=sin (3x+4)        … (2)Now,(h o (gof)) (x)=h ((gof) (x))=h(6x+4) = sin (6x+4)    [from (1)]((hogo f) (x)=(hog) (f (x))=(hog) (2x)=sin (6x+4)    [from (2)]So, (h o (gof)) (x)=((hogo f) (x), ∀xNHence, h o (gof)=(hogo f

Page No 2.46:

Question 11:

Give examples of two functions f : N → N and g : N → N, such that gof is onto but f is not onto.

ANSWER:

Let us consider a function f : N → given by f(x) = x +1 , which is not onto.
[This not onto because if we take 0 in N (co-domain), then,
0=x+1
⇒x=-1∉N]

Let us consider g : N → N  given by
g (x)={x-1, if x>11, if x=1Now, let us find (gof) (x)Case 1: x>1(gof) (x)=g (f (x))=g (x+1)=x+1-1=xCase 2: x=1(gof) (x)=g (f (x))=g (x+1)=1From case-1 and case-2, (gof) (x)=x, ∀xN, which is an identity function and, hence, it is onto.

Page No 2.46:

Question 12:

Give examples of two functions f : N → Z and g : Z → Z, such that gof is injective but g is not injective.

ANSWER:

Let f : N → be given by f (x) = x, which is injective.
(If we take f(x) = f(y), then it gives x = y)

Let g : Z → Z be given by g (x) = |x|, which is not injective.
If we take f(x) = f(y), we get:
|x| = |y|
⇒x = ± y

Now, gof N → Z.
(gof) (x)=g (f (x))=g (x)=|x|
Let us take two elements x and y in the domain of gof , such that
(gof) (x)=(gof) (y)⇒|x|=|y|⇒x=y (We don’t get ± here because x, y ∈N)
So, gof is injective.
 

Page No 2.46:

Question 13:

If f : A → B and g : B → C are one-one functions, show that gof is a one-one function.

ANSWER:

Given,  f : A → B and g : B → C are one – one.
Then, gof : A → B
Let us take two elements x and y from A, such that
(gof) (x)=(gof) (y)⇒g (f (x))=g (f (y))⇒f (x)=f (y) (As, g is one-one)⇒x=y (As, f is one-one)
Hence, gof is one-one.

Page No 2.46:

Question 14:

If f : A → B and g : B → C are onto functions, show that gof is a onto function.

ANSWER:

Given,  f : A → B and g : B → C are onto.
Then, gof : A → C
Let us take an element z in the co-domain (C).
Now, z is in C and g : B → C is onto.
So, there exists some element y in B, such that g (y) = z … (1)
Now, y is in B and f : A → is onto.
So, there exists some x in A, such that f (x) = y … (2)
From (1) and (2),
z = g (y) = g (f (x)) = (gof) (x)
So, z = (gof) (x), where x is in A.
Hence, gof is onto.

Page No 2.54:

Question 1:

Find fog and gof  if
(i) f(x)=exg(x)=loge x
(ii) f(x)=x2, g(x)=cos x
(iii) f(x)=|x|, g (x)=sin x
(iv) f(x)=x+1, g(x)=ex
(v) f(x)=sin-1 xg(x)=x2
(vi) f(x)=x+1, g(x)=sin x
(vii) f(x)=x+1, g(x)=2x+3
(viii) f(x)=cc ∈ Rg(x)=sin x2
(ix) f(x)=x2+2, g(x)=1-11-x

ANSWER:

(if (x)=exg(x)=loge xf:R→(0,∞); g:(0,∞)→RComputing fog:Clearly, the range of g is a subset of the domain of f.fog : (0,∞)→R(fog) (x)=f (g (x))=f (loge x)=loge ex=xComputing gof:Clearly, the range of f is a subset of the domain of g.⇒fog : RR(gof) (x)=g (f (x))=g (ex)=loge ex=x

(iif (x)=x2, g(x)=cos xf:R→[0, ∞) ; g:R→[-1, 1]Computing fog:Clearly, the range of g is not a subset of the domain of f.⇒Domain (fog)={xx∈domain of g and g(x)∈domain of f}⇒Domain (fog)=xxR and cos x ∈R}⇒Domain of (fog)=RfogRR(fog) (x)=f (g (x))=f (cos x)=cos2xComputing gof:Clearly, the range of f is a subset of the domain of g.⇒fog : RR(gof) (x)=g (f (x))=g (x2)=cos (x2)


(iiif (x)=|x|, g(x)=sin xf:R→(0, ∞); g:R→[-1, 1]Computing fog:Clearly, the range of g is a subset of the domain of f.⇒fog : RR(fog) (x)=f (g (x))=f (sin x)=|sin x|Computing gof:Clearly, the range of f is a subset of the domain of g.⇒fog : RR(gof) (x)=g (f (x))=g (|x|)=sin |x|

(ivf (x)=x+1, g(x)=exf:RRg:R→[1, ∞)Computing fog:Clearly, range of g is a subset of domain of f.⇒fog : RR(fog) (x)=f (g (x))=f (ex)=ex+1Computing gof:Clearly, range of f is a subset of domain of g.⇒fog : RR(gof) (x)=g (f (x))=g (x+1)=ex+1

(vf (x)=sin-1xg(x)=x2f:[-1,1]→[-π2,π2] ; g:R→[0, ∞)Computing fog:Clearly, the range of g is not a subset of the domain of f.Domain (fog)={xx∈domain of g and g(x)∈domain of f}Domain (fog)={xxR and x2∈[-1,1]}Domain (fog)={xxR and x∈[-1,1]}Domain of (fog)=[-1,1]fog: [-1,1]→R(fog) (x)=f (g (x))=f (x2)=sin-1 (x2)Computing gof:Clearly, the range of f is a subset of the domain of g.fog : [-1,1]→R(gof) (x)=g (f (x))=g (sin-1x)=(sin-1 x)2

(vi) f(x)=x+1, g(x)=sin xf:RR ; g:R→[-1, 1]Computing fog:Clearly, the range of g is a subset of the domain of f.⇒fogRR(fog) (x)=f (g (x))=f (sin x)=sin x+1Computing gof:Clearly, the range of f is a subset of the domain of g.⇒fog : RR(gof) (x)=g (f (x))=g (x+1)=sin (x+1)

(viif (x)=x+1, g(x)=2x+3f:RR ; g:RRComputing fog:Clearly, the range of g is a subset of the domain of f.⇒fogRR(fog) (x)=f (g (x))=f (2x+3)=2x+3+1=2x+4Computing gof:Clearly, the range of f is a subset of the domain of g.⇒fog : RR(gof) (x)=g (f (x))=g (x+1)=2 (x+1)+3=2x+5

(viiif (x)=cg(x)=sin x2f:R→{c} ; g:R→[0, 1]Computing fog:Clearly, the range of g is a subset of the domain of f.fogRR(fog) (x)=f (g (x))=f (sin x2)=cComputing gof:Clearly, the range of f is a subset of the domain of g.⇒fog : RR(gof) (x)=g (f (x))=g (c)=sin c2

(ixf(x)=x2+2f:R→[2,∞)  g(x)=1-11-xFor domain of g: 1-x≠0 ⇒x≠1⇒Domain of g=R-{1}g(x)=1-11-x=1-x-11-x=-x1-xFor range of g:y=-x1-xyxy=-xy=xyxy=x(y-1)⇒x=yy-1Range of g =R-{1}So, g: R-{1}→R-{1}Computing fog:Clearly, the range of g is a subset of the domain of f.⇒fogR-{1}→R(fog) (x)=f (g (x))=f (-xx-1)=(-xx-1)2+2=x2+2x2+2-4x(1-x)2=3x2-4x+2(1-x)2Computing gof:Clearly, the range of f is a subset of the domain of g.⇒gof : RR(gof) (x)=g (f (x))=g (x2+2)=1-11-(x2+2)=1-1-(x2+1)=x2+2x2+1

Page No 2.54:

Question 2:

Let f(x) = x2 + x + 1 and g(x) = sin x. Show that fog ≠ gof.

ANSWER:

(fog) (x)=f (g (x))=f(sin x)=sin2x+sin x+1and (gof) (x)=g (f (x))=g (x2+x+1)= sin (x2+x+1)So, foggof.

Page No 2.54:

Question 3:

If f(x) = |x|, prove that fof = f.

ANSWER:

Domains of  f and fof are same as R.
(fof) (x)=f (f (x))=f (|x|)=| |x| |=|x|=f (x)So,(fof) (x)=f (x), ∀xRHence, fof=f

Page No 2.54:

Question 4:

If f(x) = 2x + 5 and g(x) = x2 + 1 be two real functions, then describe each of the following functions:
(i) fog
(ii) gof
(iii) fof
(iv) f2

Also, show that fof ≠ f2

ANSWER:

f(x) and g(x) are polynomials.
→ and → R.
So, fog → R  and gof → R.
(i) (fog) (x)=f (g (x))=f (x2+1)=2 (x2+1)+5=2×2+2+5=2×2+7

(ii) (gof) (x)=g (f (x))=g (2x+5)=(2x+5)2+1=4×2+20x+26

(iii) (fof) (x)=f (f (x))=f (2x+5)=2 (2x+5)+5=4x+10+5=4x+15

(iv) f2 (x)=f(x)×f(x)=(2x+5)(2x+5)=(2x+5)2=4×2+20x+25



→→  →

Page No 2.54:

Question 5:

If f(x) = sin x and g(x) = 2x be two real functions, then describe gof and fog. Are these equal functions?
 

ANSWER:

We know thatf:R→[-1, 1] and gRRClearly, the range of f is a subset of the domain of g.gof:RR(gof) (x)=g (f (x))=g (sin x)=2sin x

Clearly, the range of g is a subset of the domain of f.fog:RRSo, (fog) (x)=f (g (x))=f (2x)=sin (2x)

Clearly, fog≠gof

Page No 2.54:

Question 6:

Let fgh be real functions given by f(x) = sin xg (x) = 2x and h (x) = cos x. Prove that fog = go (fh).

ANSWER:

We know that f:R→[-1, 1] and g:RRClearly, the range of g is a subset of the domain of f.fog:RRNow, (fh) (x)=f(x)h(x)=(sin x) (cos x)=12 sin (2x)Domain of fh is R.Since range of sin x is [-1,1],-1≤sin 2x≤1⇒-12≤sin x2≤12Range of fh  =[-12, 12]So, (fh):R→[-12, 12]Clearly, range of fh is a subset of g.⇒go(fh):RR⇒domains of fog and go(fh) are the same.So, (fog) (x)=f (g (x))=f (2x)=sin (2x)and (go(fh))(x)= g ((fh) (x))=g (sinx cos x)=2sin x cos x=sin (2x)⇒(fog) (x)=( go(fh))(x), ∀xRHence, fog = go(fh)

Page No 2.54:

Question 7:

Let  f  be any real function and let g be a function given by g(x) = 2x. Prove that gof = f + f.

ANSWER:

Given, f:RRSince g(x)=2x is a polynomial, g:RRClearly, gof:RR and f+f:RRSo, domains of gof and f+f are the same.(gof) (x)=g (f (x))=2 f(x)(f+f) (x)=f(x)+f(x)=2 f(x)⇒(gof) (x)=(f+f) (x), ∀xRHence, gof =f+f

Page No 2.54:

Question 8:

If f(x)=√1-x and g(x)=loge x are two real functions, then describe functions fog and gof.

ANSWER:

 f(x)=√1-xFor domain, 1-x≥0⇒x≤1⇒domain of f =(-∞, 1]⇒f:(-∞, 1]→(0,∞) g(x)=loge xClearly, g : (0, ∞)→RComputation of fog:Clearly, the range of g is not a subset of the domain of f.So,we need to compute the domain of fog.⇒Domain (fog)={x : x∈Domain (g) and g(x)∈Domain of f}⇒Domain (fog)={xx∈(0, ∞) and loge x ∈ (-∞, 1]}⇒Domain (fog)={x:x∈(0, ∞) and x∈ (0, e]}⇒Domain (fog)={xx ∈(0, e]}⇒Domain (fog)=(0, e]⇒fog: (0, e)→RSo, (fog) (x)=f (g (x))=f (loge x)=√1-loge x Computation of gof:Clearly, the range of f is  a subset of the domain of g.⇒gof:(-∞,1]→R⇒(gof) (x)=g (f (x))=g (√1-x)=loge√1-x=loge (1-x)12=12loge (1-x)

Page No 2.54:

Question 9:

If f:(-π2,π2)→R and g:[-1, 1]→R be defined as
f(x)=tan x and g(x)=√1-x2 respectively, describe fog and gof.

ANSWER:

g (x)=√1-x2⇒x2≥0, ∀x∈[-1, 1]⇒-x2≤0, ∀x∈[-1, 1]⇒1-x2≤1, ∀x∈[-1, 1]We know that 1-x2≥0⇒0≤1-x2≤1⇒Range of g(x)=[0, 1]So, f:(π2, π2)R and g:[1, 1] [0, 1]Computation of fog:Clearly, the range of g is a subset of the domain of f.So, fog: [-1, 1]→R(fog) (x)=f (g (x))=f (√1-x2)=tan √1-x2Computation of gof:Clearly, the range of f is not a subset of the domain of g.⇒Domain (gof)={x∈domain of f and f(x)∈domain of g}⇒Domain (gof)={x∈(π2, π2) and tan x [1,1]}⇒Domain (gof)={x∈(π2, π2) and x (π4, π4)}⇒Domain (gof)={x(π4, π4)}Now, gof:(π4, π4)RSo, (gof) (x)=g (f (x))=g (tan x)=√1-tan2x 

Page No 2.54:

Question 10:

If f(x)=√x+3 and g(x)=x2+1 be two real functions, then find fog and gof.

ANSWER:

f(x)=√x+3For domain,x+3≥0⇒x≥-3Domain of f =[-3, ∞)Since f is a square root function, range of f =[0, ∞)f: [-3, ∞)→[0, ∞)g(x)=x2+1 is a polynomial.⇒g:RRComputation of fog:Range of g  is not a subset of the domain of f.and domain (fog)={xx∈domain of g and g(x)∈domain of f(x)}⇒Domain (fog)={x:xR and  x2+1∈[-3, ∞)}⇒Domain (fog)={x:xR and  x2+1≥-3}⇒Domain (fog)={x:xR and  x2+4≥0}⇒Domain (fog)={x:xR and xR}⇒Domain (fog)=Rfog:RR(fog) (x)=f(g (x))=f (x2+1)=√x2+1+3=√x2+4Computation of gof:Range of f  is a subset of the domain of g.gof: [-3, ∞)→R⇒(gof) (x)=g (f (x))=g (√x+3)=(√x+3)2+1=x+3+1=x+4

Page No 2.54:

Question 11:

Let f be a real function given by f(x)=√x-2.
Find each of the following:
(i) fof
(ii) fofof
(iii) (fofof) (38)
(iv) f2

Also, show that fof ≠ f2 .

ANSWER:

f(x)=√x-2For domain,x-2≥0⇒x≥2Domain of f=[2,∞)Since fis a square-root function, range of f=(0,∞)So, f:[2,∞)→(0,∞)(i) fofRange of f is not a subset of the domain of f.⇒Domain(fof)={xx ∈domain of fand f(x)∈domain of f}⇒Domain(fof)={xx ∈[2,∞) and √x-2∈[2,∞)}⇒Domain(fof)={xx ∈[2,∞) and √x-2≥2}⇒Domain(fof)={xx ∈[2,∞) and  x-2≥4}⇒Domain(fof)={xx ∈[2,∞) and  x≥6}⇒Domain(fof)={xx≥6}⇒Domain(fof)=[6, ∞)fof :[6, ∞)→R(fof) (x)=f (f (x))=f (√x-2)=√√x-2-2

(ii) fofof= (fofofWe have, f:[2,∞)→(0,∞) and fof : [6, ∞)→R⇒Range of f is not a subset of the domain of fof.Then, domain((fof)of)={xx ∈domain of fand f(x)∈domain of fof}⇒Domain((fof)of)={xx ∈[2,∞) and √x-2∈[6,∞)}⇒Domain((fof)of)={xx ∈[2,∞) and √x-2≥6}⇒Domain((fof)of)={xx ∈[2,∞) and  x-2≥36}⇒Domain((fof)of)={xx ∈[2,∞) and  x≥38}⇒Domain((fof)of)={xx≥38}⇒Domain((fof)of)=[38, ∞)fof :[38,∞)→RSo, ((fof)of) (x)=(fof) (f (x))=(fof) (√x-2)=√√√x-2-2-2

(iii) We have, (fofof) (x)=√√√x-2-2-2So, (fofof) (38)=√√√38-2-2-2=√√√36-2-2=√√6-2-2=√2-2=0

(iv) We have, fof=√√x-2-2⇒f2(x)=f(xf(x)=√x-2×√x-2=x-2So, fof ≠ f2

Page No 2.55:

Question 12:

Let f(x)={1+x,0≤x≤23-x,2<x≤3. Find fof.

ANSWER:

f(x)={1+x,0≤x≤23-x,2<x≤3It can be written as,f(x)={ 1+x,0≤x≤11+x,1<x≤23-x,2<x≤3  When,0≤x≤1Then, f(x)=1+xNow when ,0≤x≤1 then ,1≤x+1≤2Then, f(f(x))=1+(1+x)=2+x   [∵1≤f(x)<2]When ,1<x≤2Then, f(x)=1+xNow when ,1<x≤2 then,2<x+1≤3Then, f(f(x))=3-(1+x)=2-x  [∵2≤f(x)<3]When ,2<x≤3Then, f(x)=3-xNow when ,2<x≤3 then ,0≤3-x<1Then, f(f(x))=1+(3-x)=4-x     [∵0≤f(x)<1]f(f(x))={ 2+x,0≤x≤12-x,1<x≤24-x,2<x≤3  

Page No 2.55:

Question 13:

If f, g : R → R be two functions defined as f(x) = |x| + x and g(x) = |x| –x,∀x∈R. Then find fog and gof. Hence find fog(–3), fog(5) and gof (–2).

ANSWER:

Given: f(x) = |x| + x 
and g(x) = |x| – x,∀x∈R

fog = f(g(x)) = |g(x)| + g(x)                           =||x| – x|+(|x| – x)

Therefore,
 f(g(x)) = {0                x≥04x             x<0fog = {4x       x<00          x≥0

gof = g(f(x))=|f(x)|-f(x)                          =||x|+x|-(|x|+x)g(f(x))={0         x≥00         x<0
Therefore, g(f(x)) = gof = 0

Now, fog(−3) =(4)(−3) = −12                 (since, fog = 4x for x < 0)

fog(5) = 0                                                (since, fog = 0 for x ≥ 0)

gof(−2) = 0                                               (since, gof = 0 for x < 0)

Page No 2.68:

Question 1:

State with reasons whether the following functions have inverse:
(i) f : {1, 2, 3, 4} → {10} with f = {(1, 10), (2, 10), (3, 10), (4, 10)}
(ii) g : {5, 6, 7, 8} → {1, 2, 3, 4} with g = {(5, 4), (6, 3), (7, 4), (8, 2)}
(iii) h : {2, 3, 4, 5} → {7, 9, 11, 13} with h = {(2, 7), (3, 9), (4, 11), (5, 13)}

ANSWER:

(i) f : {1, 2, 3, 4} → {10} with f = {(1, 10), (2, 10), (3, 10), (4, 10)}
We have:
(1) = (2) = (3) = (4) = 10
is not one-one.
is not a bijection.
So, f does not have an inverse.

(ii) g : {5, 6, 7, 8} → {1, 2, 3, 4} with g = {(5, 4), (6, 3), (7, 4), (8, 2)}
(5) = (7) = 4
is not one-one.
is not a bijection.
So, f does not have an inverse.

(iii) h : {2, 3, 4, 5} → {7, 9, 11, 13} with h = {(2, 7), (3, 9), (4, 11), (5, 13)}
Here, different elements of the domain have different images in the co-domain.
is one-one.
Also, each element in the co-domain has a pre-image in the domain.
is onto.
is a bijection.
has an inverse and it is given by
h-1={(7, 2), (9, 3), (11, 4), (13, 5)}

Page No 2.68:

Question 2:

Find f −1 if it exists : f : A → B, where
(i) A = {0, −1, −3, 2}; B = {−9, −3, 0, 6} and f(x) = 3 x.
(ii) A = {1, 3, 5, 7, 9}; B = {0, 1, 9, 25, 49, 81} and f(x) = x2

ANSWER:

(i) A = {0, −1, −3, 2}; B = {−9, −3, 0, 6} and f(x) = 3 x.
Given: f(x) = 3 x
So,  f = {(0, 0), (-1, -3), (-3, -9), (2, 6)}
Clearly, this is one-one.
Range of f = Range of =B
So, is a bijection and, thus, f -1 exists.  
Hence, f -1= {(0, 0), (-3, -1), (-9, -3), (6, 2)}

(ii) A = {1, 3, 5, 7, 9}; B = {0, 1, 9, 25, 49, 81} and f(x) = x2
Given: f(x) = x2
So, f = {(1, 1), (3, 9), (5, 25), (7,49), (9, 81)}
Clearly, is one-one.
But this is not onto because the element 0 in the co-domain (B) has no pre-image in the domain (A) .
is not a bijection.
So, f -1does not exist.

Page No 2.68:

Question 3:

Consider f : {1, 2, 3} → {abc} and g : {abc} → {apple, ball, cat} defined as f (1) = af (2) = bf (3) = cg (a) = apple, g (b) = ball and g (c) =  cat. Show that fg and gof are invertible. Find f−1g−1 and gof−1 and show that (gof)−1 = f 1o g−1.

ANSWER:

f={(1, a), (2, b), (3, c)} and g={(a, apple), (b, ball), (c, cat)}Clearly, f and g are bijections.So, f and g are invertible.Now,f-1={(a, 1), (b, 2), (c, 3)} and g-1={(apple, a), (ball, b), (cat, c)}So, f-1o g-1={(apple, 1), (ball, 2), (cat, 3)}         …(1)f:{1, 2, 3}→{a, b, c} and g:{a, b, c}→{apple, ball, cat}So, gof:{1, 2, 3}→{apple, ball, cat}⇒(gof) (1)=g (f (1))=g (a)=apple(gof) (2)=g (f (2))=g (b)=ball,and (gof) (3)=g (f (3))=g (c)=cat∴gof ={(1, apple), (2, ball), (3, cat)}Clearly, gofis a bijection.So, gof is invertible.(gof)-1={(apple, 1), (ball, 2), (cat, 3)}          …(2)From (1) and (2), we get:(gof)-1=f-1o g-1

Page No 2.68:

Question 4:

Let A = {1, 2, 3, 4}; B = {3, 5, 7, 9}; C = {7, 23, 47, 79} and f : A → Bg : B → C be defined as f(x) = 2x + 1 and g(x) = x2 − 2. Express (gof)−1 and f−1 og−1 as the sets of ordered pairs and verify that (gof)−1 = f−1 og−1.

ANSWER:

 f(x)=2x+1⇒f={(1, 2(1)+1), (2, 2(2)+1), (3, 2(3)+1), (4, 2(4)+1)}={(1, 3), (2, 5), (3, 7), (4, 9)}g(x)=x2-2⇒g={(3, 32-2), (5, 52-2), (7, 72-2), (9, 92-2)}={(3, 7), (5, 23), (7, 47), (9, 79)}Clearly and g are bijections and, hence, f-1:BA and g-1: CB exist.So, f-1={(3, 1), (5, 2), (7, 3), (9, 4)} and g-1={(7, 3), (23, 5), (47, 7), (79, 9)}Now, (f-1 o g-1):CAf-1 o g-1={(7, 1), (23, 2), (47, 3), (79, 4)}        …(1)Also, f:AB and g:BC,⇒gof:AC, (gof)-1:CASo, f-1 o g-1and (gof)-1 have same domains.(gof)(x)=g (f (x))=g (2x+1)=(2x+1)2-2⇒ (gof)(x)=4x2+4x+1-2⇒ (gof)(x)=4x2+4x-1Then, (gof)(1)=g (f (1))=4+4-1=7,(gof)(2)=g (f (2))=4+4-1=23,(gof)(3)=g (f (3))=4+4-1=47 and (gof)(4)=g (f (4))=4+4-1=79So, gof={(1, 7), (2, 23), (3, 47), (4, 79)}⇒(gof)-1={(7, 1), (23, 2), (47, 3), (79, 4)}     …(2)From (1) and (2), we get: (gof)-1=f-1 o g-1

Page No 2.68:

Question 5:

Show that the function f : Q → Q, defined by f(x) = 3x + 5, is invertible. Also, find f−1

ANSWER:

Injectivity of f:
Let and be two elements of the domain (Q), such that
f(x)=f(y)
⇒3x + 5 =3y + 5
⇒3x = 3y
x = y
So, is one-one.

Surjectivity of f:
Let y be in the co-domain (Q), such that f(x) = y

⇒3x+5=y⇒3x=y-5⇒x=y-53∈Q (domain)


f is onto.
So, f is a bijection and, hence, it is invertible.

Finding f  –1:
Let f-1(x)=y                    …(1)⇒x=f(y)⇒x=3y+5⇒x-5=3yy=x-53So, f-1(x)=x-53       [from (1)]

Page No 2.68:

Question 6:

Consider f : R → R given by f(x) = 4x + 3. Show that f is invertible. Find the inverse of f.

ANSWER:

Injectivity of f :
Let and be two elements of domain (R), such that
f(x) = f(y)
⇒4x + 3 = 4y + 3
⇒4x = 4y
⇒x = y
So, is one-one.

Surjectivity of f :
Let y be in the co-domain (R), such that f(x) = y.

⇒4x+3=y⇒4x=y- 3⇒x=y- 34∈R(Domain)


f is onto.
So, f is a bijection and, hence, is invertible.

Finding f  –1:
Let f-1(x)=y                   …(1)⇒x=f(y)⇒x=4y+3⇒x-3=4yy=x-34So, f-1(x)=x-34       [from (1)]

Page No 2.68:

Question 7:

Consider f : R → R+ → [4, ∞) given by f(x) = x2 + 4. Show that f is invertible with inverse f−1 of f given by f−1 (x)=√x-4, where R+ is the set of all non-negative real numbers.

ANSWER:

Injectivity of f :
Let and be two elements of the domain (Q), such that
f(x)=f(y)
x2+4=y2+4⇒x2=y2⇒x=y   (as co-domain as R+)
So, is one-one.

Surjectivity of f :
Let y be in the co-domain (Q), such that f(x) = y

⇒x2+4=y⇒x2=y-4⇒x=√y-4∈R


f is onto.
So, f is a bijection and, hence, it is invertible.

Finding f  –1:
Let f-1(x)=y                    …(1)⇒x=f(y)⇒x=y2+4⇒x-4=y2⇒y=√x-4So, f-1(x)=√x-4          [from (1)]

Page No 2.68:

Question 8:

If f(x)=4x+36x-4, x≠23, show that fof(x) = x for all x≠23. What is the inverse of f?

ANSWER:

(fof)(x)=f(f(x))=f (4x+36x-4)=4(4x+36x-4)+36(4x+36x-4)-4=16x+12+18x-1224x+18-24x+16=34x34=x⇒(fof)(x)=x=IX, where I is an identity function.So, f=f-1 Hence, f-1=4x+36x-4

Page No 2.68:

Question 9:

Consider f : R+ → [−5, ∞) given by f(x) = 9x2 + 6x − 5. Show that f is invertible with f-1(x)=√x+6-13.

ANSWER:

Injectivity of :
Let and be two elements of domain (R+), such that
f(x)=f(y)
⇒9x2+6x-5=9y2+6y-5⇒9x2+6x=9y2+6yx=y (As, xyR+)
So, is one-one.

Surjectivity of f:
Let y is in the co domain (Q) such that f(x) = y

⇒9×2+6x-5=y⇒9×2+6x=y+5⇒9×2+6x+1=y+6 (Adding 1 on both sides)⇒(3x+1)2=y+6⇒3x+1=√y+6⇒3x=√y+6-1⇒x=√y+6-13∈R+(domain)


f is onto.
So, f is a bijection and hence, it is invertible.

Finding f  –1:
Let f-1(x)=y                          …(1)⇒x=f(y)⇒x=9y2+6y-5⇒x+5=9y2+6yx+6=9y2+6y+1       (adding 1 on both sides)⇒x+6=(3y+1)2⇒3y+1=√x+6⇒3y=√x+6-1⇒y=√x+6-13So, f-1(x)=√x+6-13     [from (1)]

Page No 2.69:

Question 10:

If f : R → R be defined by f(x) = x3 −3, then prove that f−1 exists and find a formula for f−1. Hence, find f−1 (24) and f−1 (5).

ANSWER:

Injectivity of :
Let and be two elements in domain (R),
 
such that,  x3-3=y3-3            ⇒x3=y3            ⇒x=y
So, is one-one.

Surjectivity of f :
Let y be in the co-domain (R) such that f(x) = y

⇒x3-3=y⇒x3=y+3⇒x=3√y+3∈R


f is onto.
So, f is a bijection and, hence, it is invertible.

Finding f  –1:
Let f-1(x)=y                         …(1)⇒x=f(y)⇒x=y3-3⇒x+3=y3⇒y=3√x+3 = f-1(x)       [from (1)]So, f-1(x)=3√x+3 Now, f-1(24)=3√24+3=3√27=3√33=3 and f-1(5)=3√5+3=3√8=3√23=2 

Page No 2.69:

Question 11:

A function f : R → R is defined as f(x) = x3 + 4. Is it a bijection or not? In case it is a bijection, find f−1 (3).

ANSWER:

Injectivity of f:
Let and be two elements of domain (R), such that
f(x)=f(y)⇒x3+4=y3+4⇒x3=y3⇒x=y
So, is one-one.

Surjectivity of f:
Let y be in the co-domain (R), such that f(x) = y.

⇒x3+4=y⇒x3=y-4⇒x=3√y-4∈R (domain)


⇒ f is onto.
So, f is a bijection and, hence, is invertible.

Finding f  –1:
Let f-1(x)=y              …(1)⇒x=f(y)⇒x=y3+4⇒x-4=y3⇒y=3√x-4So, f-1(x)=3√x-4       [from (1)]f-1(3)=3√3-4 =3√-1=-1

Page No 2.69:

Question 12:

If f : Q → Qg : Q → Q are two functions defined by f(x) = 2 x and g(x) = x + 2, show that f and g are bijective maps. Verify that (gof)−1 = f−1 og −1.

ANSWER:

Injectivity of f:
Let and be two elements of domain (Q), such that
f(x) = f(y)
⇒2x = 2y
x = y
So, is one-one.

Surjectivity of f:
Let y be in the co-domain (Q), such that f(x) = y.

⇒2x= y⇒x= y2∈Q   (domain)


⇒ f is onto.
So, f is a bijection and, hence, it is invertible.

Finding f  –1:
Let f-1(x)=y             …(1)⇒x=f(y)⇒x=2yy=x2  So, f-1(x)=x2        (from (1))

Injectivity of g:
Let and be two elements of domain (Q), such that
g(x) = g(y)
⇒x + 2 = y + 2
⇒x = y
So, is one-one.

Surjectivity of g:
Let y be in the co domain (Q), such that g(x) = y.

⇒x+2=y⇒x= 2-y∈Q   (domain)


⇒ g is onto.
So, g is a bijection and, hence, it is invertible.

Finding g –1:
Let g-1(x)=y             …(2)⇒x=g(y)⇒x=y+2⇒y=x-2So, g-1(x)=x-2     (From (2))

Verification of (gof)−1 = f−1 og −1:

f(x)=2xg(x)=x+2and f-1(x)=x2; g-1(x)=x-2Now, (f-1o g-1)(x)=f-1(g-1(x))⇒(f-1o g-1)(x)=f-1(x-2)⇒(f-1o g-1)(x)=x-22      …(3)(gof)(x)=g (f (x))=g (2x)=2x+2Let (gof)-1(x)=y  ….   (4)x=(gof)(y)⇒x=2y+2⇒2y=x-2⇒y=x-22 ⇒(gof)-1(x)=x-22   [from (4)   … (5)]From (3) and (5), (gof)-1=f-1o g-1

Page No 2.69:

Question 13:

Let A = R – {3} and B = R – {1}. Consider the function f : A → B defined by f(x) = x-2x-3. Show that f is one-one and onto and
hence find f-1.                                                                                                                                                                        [CBSE 2012, 2014]

ANSWER:

We have,

A = R – {3} and B = R – {1}
The function f : A → B defined by f(x) = x-2x-3

Let x,yA such that f(x)=f(y). Then,x-2x-3=y-2y-3⇒xy-3x-2y+6=xy-2x-3y+6⇒-x=-yx=y∴ f is one-one.Let yB. Then, y≠1.The function f is onto if there exists xA such that f(x)=y.Now,f(x)=yx-2x-3=yx-2=xy-3yxxy=2-3yx(1-y)=2-3yx=2-3y1-yA    [y≠1]Thus, for any yB, there exists 2-3y1-yA such thatf(2-3y1-y)=(2-3y1-y)-2(2-3y1-y)-3=2-3y-2+2y2-3y-3+3y=-y-1=y∴ f is onto.So, f is one-one and onto fucntion.Now,As, x=(2-3y1-y)So, f-1(x)=(2-3x1-x)=3x-2x-1

Page No 2.69:

Question 14:

Consider the function f : R+ →[-9,∞] given by f(x) = 5x2 + 6x – 9. Prove that f is invertible with f-1(y) = √54+5y-35.      [CBSE 2015]

ANSWER:

We have,f(x)=5x2+6x-9Let y=5x2+6x-9=5(x2+65x-95)=5(x2+2×x×35+925-925-95)=5((x+35)2-925-95)=5(x+35)2-95-9=5(x+35)2-545⇒y+545=5(x+35)2⇒5y+5425=(x+35)2⇒√5y+5425=x+35⇒x=√5y+545-35⇒x=√5y+54-35Let g(y)=√5y+54-35Now,fog(y)=f(g(y))=f(√5y+54-35)=5(√5y+54-35)2+6(√5y+54-35)-9=5(5y+54+9-6√5y+5425)+(6√5y+54-185)-9=5y+63-6√5y+545+6√5y+54-185-9=5y+63-18-455=y=IY, Identity functionAlso, gof(x)=g(f(x))=g(5x2+6x-9)=√5(5x2+6x-9)+54-35=√25x2+30x-45+54-35=√25x2+30x+9-35=√(5x+3)2-35=5x+3-35=x=IX, Identity functionSo, f is invertible.Also, f-1(y)=g(y)=√5y+54-35

Page No 2.69:

Question 15:

 Let f : N→N be a function defined as f (x)=9×2+6x-5. Show that : N→S, where  is the range of f, is invertible. find the inverse of and hence find f -1(43) and -1(163).

ANSWER:

We have,

f : N→N is a function defined as (x) = 9x2 + 6– 5.

Let y = (x) = 9x2 + 6– 5

y=9x2+6x-5⇒y=9x2+6x+1-1-5⇒y=(9x2+6x+1)-6⇒y=(3x+1)2-6⇒y+6=(3x+1)2

⇒√y+6=3x+1             (∵yN)⇒√y+6-1=3xx=√y+6-13⇒g(y)=√y+6-13         [Let x=g(y)]

Now,

fog(y)=f[g(y)]=f(√y+6-13)=9(√y+6-13)2+6(√y+6-13)-5=9(y+6-2√y+6+19)+2(√y+6-1)-5=y+6-2√y+6+1+2√y+6-2-5=y=IY, Identity function

gof(x)=g[f(x)]=g(9x2+6x-5)=√(9x2+6x-5)+6-13=√(9x2+6x+1)-13=√(3x+1)2-13=(3x+1)-13=3x3=x=IX, Identity function

Since, fog(y) and gof(x) are identity function.

Thus, f is invertible.

So, f-1(x)=g(x)=√x+6-13.

Now,

-1(43) = √43+6-13=√49-13=7-13=63=2

And -1(163) = √163+6-13=√169-13=13-13=123=4

Page No 2.69:

Question 16:

Let f  -{-43} →R be a function defined as f(x) =4x3x+4 . Show that
     f : R -{-43} → Rang (f) is one-one and onto. Hence, find -1.

ANSWER:

The function f:R-{-43}→R-{43} is given by f(x)=4x3x+4.
Injectivity: Let xyR-{-43} be such that
f(x)=f(y)⇒4x3x+4=4y3y+4⇒4x(3y+4)=4y(3x+4)⇒12xy+16x=12xy+16y⇒16x=16yx=y
Hence, is one-one function.
Surjectivity: Let y be an arbitrary element of R-{43}. Then,
f(x) = y
⇒4x3x+4=y⇒4x=3xy+4y⇒4x-3xy=4yx=4y4-3y
As yR-{43}, 4y4-3yR.
Also, 4y4-3y≠-43 because 4y4-3y=-43⇒12y=-16+12y⇒0=-16, which is not possible.
Thus,
x=4y4-3yR-{-43} such that
f(x)=f(4x3x+4)=4(4y4-3y)3(4y4-3y)+4=16y12y+16-12y=16y16=y, so every element in R-{43} has pre-image in R-{-43}.
Hence, f is onto.
Now,
x=4y4-3y
Replacing x by f-1(x) and y by x, we have
 f-1(x)=4x4-3x   

Page No 2.69:

Question 17:

Let A = R – {2} and B = R – {1}. If f : A → B is a function defined by f(x)=x-1x-2, show that f is one-one and onto. Find f1.

ANSWER:

Given: f(x)=x-1x-2

To show f is one-one:

Let f(x1)=f(x2)⇒x1-1x1-2=x2-1x2-2⇒(x1-1)(x2-2)=(x2-1)(x1-2)⇒x1x2-2x1-x2+2=x1x2-2x2-x1+2⇒-2x1-x2=-2x2-x1⇒-2x1+x1=-2x2+x2⇒-x1=-x2⇒x1=x2Hence, f is one-one.To show f is onto: Let yB∴ y=f(x)⇒y=x-1x-2⇒y(x-2)=x-1⇒xy-2y=x-1⇒xyx=2y-1⇒x(y-1)=2y-1⇒x=2y-1y-1Thus, for every value of y in R-{1}, there exists a pre-image x=2y-1y-1 in R-{2}.Hence, f is onto.

Since, is one-one and onto
Therefore, is invertible with f-1(y)=2y-1y-1.

Hence, f-1(x)=2x-1x-1.

 

Page No 2.69:

Question 18:

Show that the function N → N defined by f(x) = x2 + x + 1 is one-one but not onto. Find the inverse of N → S, where S is range of f.

ANSWER:

Given: The function N → N defined by f(x) = x2 + x + 1

To show f is one-one:

Let f(x1)=f(x2)⇒x21+x1+1=x22+x2+1⇒x21+x1=x22+x2⇒x21+x1-x22-x2=0⇒x21-x22+x1-x2=0⇒(x1-x2)(x1+x2)+(x1-x2)=0⇒(x1-x2)(x1+x2+1)=0⇒x1-x2=0 or x1+x2+1=0⇒x1=x2 or x1=-(x2+1)⇒x1=x2         (∵ x1, x2∈N)Hence, f is one-one.To show f is not onto: Since f(x)=x2+x+1∴ f(1)=3f(2)=7f(3)=13and so onThus, Range of f={3, 7, 13, …}≠NHence, f is not onto.Now, Let f:N→Range of fy=x2+x+1⇒x2+x+1-y=0⇒x2+x+(1-y)=0⇒x=-1±√12-4(1)(1-y)2(1)⇒x=-1±√1-4+4y2⇒x=-1±√4y-32⇒x=-1+√4y-32 or x=-1-√4y-32⇒x=-1+√4y-32        (∵ xN)Hence, f-1(x)=-1+√4x-32.


 

Page No 2.69:

Question 19:

Let f : [−1, ∞) → [−1, ∞) be given by f(x) = (x + 1)2 − 1, x ≥ −1. Show that f is invertible. Also, find the set S = {x : f(x) = f−1 (x)}.

ANSWER:

Injectivity: Let x and y ∈[-1, ∞), such that f(x)=f(y)⇒(x+1)2-1=(y+1)2-1⇒(x+1)2=(y+1)2⇒(x+1)=(y+1)⇒x=ySo, f is a injection.Surjectivity: Let y ∈[-1, ∞). Then, f(x)=y⇒(x+1)2-1=y⇒x+1=√y+1⇒x=√y+1-1Clearly, x=√y+1-1 is real for all y≥-1.Thus, every element y ∈[-1, ∞) has its pre-image x∈[-1, ∞) given by x=√y+1-1.⇒f is a surjection.So, f is a bijection.Hence, f is invertible.Let f-1(x)=y                          …(1)⇒f(y)=x⇒(y+1)2-1=x⇒(y+1)2=x+1⇒y+1=√x+1⇒y=±√x+1-1⇒f-1(x)=±√x+1-1   [from (1)]f(x)=f-1(x)⇒(x+1)2-1=±√x+1-1⇒(x+1)2=±√x+1⇒(x+1)4=x+1⇒(x+1)[(x+1)3-1]=0⇒x+1=0 or (x+1)3-=0⇒x=-1 or (x+1)3=1⇒x=-1 or x+1=1⇒x=-1 or x=0⇒S={0, -1}

Page No 2.69:

Question 20:

Let A = {x &epsis; R | −1 ≤ x ≤ 1} and let f : A → Ag : A → A be two functions defined by f(x) = x2 and g(x) = sin (π x/2). Show that g−1 exists but f−1 does not exist. Also, find g−1.

ANSWER:

 f is not one-one because
f(-1)=(-1)2=1and f(1)=12=1
⇒ -1 and 1 have the same image under f.
⇒ f is not a bijection.
So, f -1 does not exist.

Injectivity of g:
Let x and y be any two elements in the domain (A), such that
g(x)=g(y)⇒sin (πx2)=sin (πy2)⇒(πx2)=(πy2)⇒x=y
So, is one-one.

Surjectivity of g:
Range of g = [sin (π(-1)2), sin (π(1)2)] = [sin (-π2), sin (π2)] = [-1, 1] = (co-domain of g)
g is onto.
g is a bijection.
So, g-1 exists.

Also,
let g-1(x)=y                      …(1)⇒g(y)=x⇒sin(πy2)=x⇒(πy2)=sin-1 xy=2πsin-1 xg-1(x)=2πsin-1 x        [from (1)]

Page No 2.69:

Question 21:

Let f be a function from R to R, such that f(x) = cos (x + 2). Is f invertible? Justify your answer.

ANSWER:

Injectivity:
Let and y be two elements in the domain (R), such that
f(x)=f(y)⇒cos(x+2)=cos(y+2)⇒x+2=y+2 or x+2=2π-(y+2)⇒x=or x+2=2π-y-2⇒x=or x=2π-y-4So, we cannot say that x=yFor example,cosπ2=cos 3π2=0So,π2 and 3π2  have the same image 0.
⇒ is not one-one.
 f is not a bijection.
Thus, f  is not invertible.

Page No 2.69:

Question 22:

If A = {1, 2, 3, 4} and B = {abcd}, define any four bijections from A to B. Also give their inverse functions.

ANSWER:

f1={(1, a), (2, b), (3, c), (4, d)}⇒f1-1={(a, 1), (b, 2), (c, 3), (d, 4)}f2={(1, b), (2, a), (3, c), (4, d)}⇒f2-1={(b, 1), (a, 2), (c, 3), (d, 4)}f3={(1, a), (2, b), (4, c), (3, d)}⇒f3-1={(a, 1), (b, 2), (c, 4), (d, 3)}f4={(1, b), (2, a), (4, c), (3, d)}⇒f4-1={(b, 1), (a, 2), (c, 4), (d, 3)}
Clearly, all these are bijections because they are one-one and onto.

Page No 2.69:

Question 23:

Let A and B be two sets, each with a finite number of elements. Assume that there is an injective map from A to B and that there is an injective map from B to A. Prove that there is a bijection from A to B.

ANSWER:

 A and B are two non empty sets. Let f be a function from A to B .It is given that there is injective map from A to B. That means f is one-one function .It is also given that there is injective map from  B to A .That means every element of set B has its image in set A.⇒f is onto function or surjective.∴ f is bijective.(If a function is both injective and surjective, then the function is bijective.)     

Page No 2.69:

Question 24:

If f : A → Ag : A → A are two bijections, then prove that
(i) fog is an injection
(ii) fog is a surjection

ANSWER:

Given: A → Ag : A → A are two bijections.
Then,  fog : A → A

(i) Injectivity of fog:
Let x and y be two elements of the domain (A), such that
(fog)(x)=(fog)(y)⇒f(g(x))=f(g(y))⇒g(x)=g(y) (As, f is one-one)⇒x=y          (As, g is one-one)
So,  fog is an injection.

(ii) Surjectivity of fog:
Let z be an element in the co-domain of fog (A).
Now, z∈A (co-domain of f) and f is a surjection.So, z=f(y), where y∈A (domain of f) ...(1)Now, y∈A (co-domain of g) and g is a surjection.Soy=g(x), where x∈A (domain of g) ...(2)From (1) and (2),z=f(y)=f(g(x))=(fog)(x), where xA(domain of fog)
So,  fog is a surjection.

Page No 2.72:

Question 1:

Let A={x ∈ R : -1 ≤x≤1}=B and C={x ∈ R : x≥0} and
let S={(xy) ∈ A×B : x2+y2=1} and S0={(xy) ∈ A×C : x2+y2=1}. Then,
(a) S defines a function from A to B
(b) S0 defines a function from A to C
(c) S0 defines a function from A to B
(d) S defines a function from A to C

ANSWER:

(a) S defines a function from A to B

Let xA⇒-1≤x≤1Now, x2+y2=1⇒y2=1-x2⇒y=±√1-x2⇒-1≤y≤1∴ yBThus, S defines a function from A to B.

Page No 2.72:

Question 2:

f : RR given by f(x)=x+√x2 is
(a) injective
(b) surjective
(c) bijective
(d) None of these

ANSWER:

f(x)=x+√x2=x±x=0 or 2x⇒ Each element of the domain has 2 images.
f is not a function.
So, the answer is (d).

Page No 2.72:

Question 3:

If f : AB given by 3f(x)+2-x=4 is a bijection, then
(a) A={x ∈ R : -1 < x < ∞}, B={x ∈ R : 2 < x < 4}
(b) A={x ∈ R : -3 < x < ∞}, B={x ∈ R : 2 < x < 4}
(c) A={x ∈ R : -2 < x < ∞}, B={x ∈ R : 2 < x < 4}
(d) None of these

ANSWER:

(d) None of these

f:AB3f(x)+2-x=4 ⇒3f(x)=4-2-xTaking log on both the sides , f(x) log 3=log (4-2-x)⇒f(x)=log (4-2-x)log 3Logaritmic function will only be defined if 4-2-x>0⇒4>2-x⇒22>2-x⇒2>-x⇒-2<xx∈ (-2,∞)That means  A={xR:-2<x<∞}As we know that, f(x)=log (4-2-x)log 3We take x=0∈ (-2,∞)⇒f(x)=1 which does not belong to any of the options .

Page No 2.72:

Question 4:

The function f : R → R defined by f(x)=2x+2|x| is
(a) one-one and onto
(b) many-one and onto
(c) one-one and into
(d) many-one and into

ANSWER:

(d) many-one and into

Graph for the given function is as follows.


A line parallel to X axis is cutting the graph at two different values.

Therefore, for two different values of x we are getting the same value of y.
That means it is many one function.

From the given graph we can see that the range is [2,∞) 
and R is the co-domain of the given function.
Hence, Co-domain≠Range
Therefore, the given function is into.



 

Page No 2.72:

Question 5:

Let the function f : R-{-b}→R-{1} be defined by

f(x)=x+ax+bab. Then,

(a) f is one-one but not onto
(b) f is onto but not one-one
(c) f is both one-one and onto
(d) None of these

ANSWER:

(c) f is both one-one and onto

Injectivity:
Let x and y be two elements in the domain R- {-b}, such that
f(x)=f(y)⇒x+ax+b=y+ay+b⇒(x+a)(y+b)=(x+b)(y+a)⇒xy+bx+ay+ab=xy+ax+by+abbx+ay=ax+by⇒(ab)x=(ab)yx=y
So, f is one-one.

Surjectivity:
Let be an element in the co-domain of f, i.e. R-{1}, such that f (x)=y

f(x)=yx+ax+b=yx+a=yx+ybxyx=ybax(1-y)=ybax=yba1-yR-{-b}
So,  is onto.

Page No 2.73:

Question 6:

The function f : AB defined by f(x)=-x2+6x-8 is a bijection if
(a) A=(-∞, 3] and B=(-∞, 1]
(b) A=[-3, ∞) and B=(-∞, 1]
(c) A=(-∞, 3] and B=[1, ∞)
(d) A=[3, ∞) and B=[1, ∞)

ANSWER:

(a) A=(-∞, 3] and B=(-∞, 1]


f(x)=-x2+6x-8 , is a polynomial functionAnd the domain of polynomial function is real number.∴xR

f(x) =-x2+6x-8       =-(x2-6x+8)       =-(x2-6x+9-1)       =-(x-3)2+1Maximum value of -(x-3)2 woud be 0∴Maximum value of -(x-3)2+1 woud be 1∴ f(x) ∈(-∞,1]



We can see from the given graph that function is symmetrical about x=3& the given function is bijective .So, x would be either (-∞,3] or [3,∞)The correct option which satisfy A and B both is: 
A=(-∞, 3] and B=(-∞, 1]

Page No 2.73:

Question 7:

Let A={x ∈ R : -1≤x≤1}=B. Then, the mapping f : AB given by f(x)=x|x| is
(a) injective but not surjective
(b) surjective but not injective
(c) bijective
(d) none of these

ANSWER:

Injectivity:
Let x and y be any two elements in the domain A.

Case-1: Let x and y be two positive numbers, such that
f(x)=f(y)⇒x|x|=y|y|⇒x(x)=y(y)⇒x2=y2⇒x=y

Case-2: Let x and y be two negative numbers, such that
f(x)=f(y)⇒x|x|=y|y|⇒x(-x)=y(-y)⇒-x2=-y2⇒x2=y2⇒x=y

Case-3: Let be positive and y be negative.
Then, xyf(x)=x|x| is positive and f(y)=y|y| is negative⇒f(x)≠f(y)So, x≠yf(x)≠f(y)
From the 3 cases, we can conclude that  f is one-one.

Surjectivity:
Let y be an element in the co-domain, such that y = f (x)
Case-1: Let y>0. Then, 0<y≤1⇒y=f(x)=x|x|>0⇒x>0⇒|x|=xf(x)=yx|x|=yx(x)=yx2=yx=√y ∈ A (We do not get ± because x>0)Case-2: Let y<0. Then, -1≤y<0⇒y=f(x)=x|x|<0⇒x<0⇒|x|=-xf(x)=yx|x|=yx(-x)=y⇒-x2=yx2=-yx=-√-y ∈ A (We do not get ± because x>0)
is onto.
is a bijection.
So, the answer is (c).

Page No 2.73:

Question 8:

Let f : RR be given by f(x)=[x2]+[x+1]-3 where [x] denotes the greatest integer less than or equal to x. Then, f(x) is
(a) many-one and onto
(b) many-one and into
(c) one-one and into
(d) one-one and onto

ANSWER:

(b) many-one and into

f : RR

f(x)=[x2]+[x+1]-3

It is many one function because in this case for two different values of x
we would get the same value of f(x) .
For x=1.1, 1.2 ∈Rf(1.1)=[(1.1)2] +[1.1+1]-3           =[1.21]+[2.1]-3           =1+2-3           =0f(1.1)=[(1.2)2] +[1.2+1]-3           =[1.44]+[2.2]-3           =1+2-3           =0


It is into function because for the given domain we would only get the integral values of
f(x).
but R is the codomain of the given function.
That means , Codomain≠Range
Hence, the given function is into function.

Therefore, f(x) is many one and into

Page No 2.73:

Question 9:

Let M be the set of all 2 × 2 matrices with entries from the set R of real numbers. Then, the function f : M → R defined by f(A) = |A| for every A ∈ M, is
(a) one-one and onto
(b) neither one-one nor onto
(c) one-one but-not onto
(d) onto but not one-one

ANSWER:


M={A=[abcd]: abcdR }fMR is given by f(A)=|A|

Injectivity:
f([0000])=|0000|=0and f([1000])=|1000|=0⇒f([0000])=f([1000])=0
So, f is not one-one.

Surjectivity:
Let y be an element of the co-domain, such that
f(A)=-yA=[abcd]⇒|abcd|=yadbc=yabcdR ⇒A=[abcd]∈M
f is onto.
So, the answer is (d).

Page No 2.73:

Question 10:

The function f : [0, ∞)→R given by f(x)=xx+1 is
(a) one-one and onto
(b) one-one but not onto
(c) onto but not one-one
(d) onto but not one-one

ANSWER:

Injectivity:
Let x and y be two elements in the domain, such that
f(x)=f(y)⇒xx+1=yy+1⇒xy+x=xy+yx=y
So, f is one-one.

Surjectivity:
Let y be an element in the co domain R, such that
y=f(x)⇒y=xx+1⇒xy+y=xx(y-1)=-yx=-yy-1Range of f=R-{1}≠co domain (R)
f is not onto.
So, the answer is (b).

Page No 2.73:

Question 11:

The range of the function f(x)=7-xPx-3 is
(a) {1, 2, 3, 4, 5}
(b) {1, 2, 3, 4, 5, 6}
(c) {1, 2, 3, 4}
(d) {1, 2, 3}

ANSWER:

We know that
7-x>0; x-3 ≥0 and 7-xx-3⇒x<7; x≥3 and 2x≤10⇒x<7; x≥3 and x≤5So, x={3, 4, 5}Range of f={P(33)(73), P(43)(74 ), P(75)(53)}={4P0, 3P1, 2P2}={1, 3, 2}={1, 2, 3}
So, the answer is (d).

Page No 2.73:

Question 12:

A function f  from the set of natural numbers to integers defined by

f(n)={n-12, when n is odd-n2, when n is evenis

(a) neither one-one nor onto
(b) one-one but not onto
(c) onto but not one-one
(d) one-one and onto both

ANSWER:

(d) one-one and onto both

Injectivity:

Let x and y be any two elements in the domain (N).
Case-1: Both x and y are even.Let f(x)=f(y)⇒-x2=-y2⇒-x=-yx=yCase-2: Both x and y are odd.Let f(x)=f(y)⇒x-12=y-12⇒x-1=y-1⇒x=yCase-3: Let x be even and y be odd. Then, f(x)=-x2and f(y)=y-12Then, clearly xy ⇒f(x)≠f(y)From all the cases, f is one-one.

Surjectivity:

Co-domain of f=Z={…,-3, -2, -1, 0, 1, 2, 3, ….}Range of f={…, -3-12, -(-2)2,-1-12, 02, 1-12, -22, 3-12, …}⇒Range of f={…,-2, 1, -1, 0, 0, -1, 1,…}⇒Range of f={…, -2, -1, 0, 1, 2, ….}⇒Co-domain of f=Range of f
⇒ f is onto.

Page No 2.73:

Question 13:

Let f be an injective map with domain {xyz} and range {1, 2, 3}, such that exactly one of the following statements is correct and the remaining are false.

f(x)=1, f(y)≠1, f(z)≠2.

The value of f-1 (1) is
(a) x
(b) y
(c) z
(d) none of these

ANSWER:

Case-1: Let f(x)=1 be true.Then, f(y)≠1 and f(z)≠2 are false.So, f(y)=1 and f(z)=2⇒ f(x)=1, f(y)=1⇒x and y have the same images.This contradicts the fact that fis one-one.Case-2: Let f(y)≠1 be true.Then, f(x)=1 and f(z)≠2 are false.So,  f(x)≠1 and f(z)=2⇒ f(x)≠1, f(y)≠1 and f(z)=2⇒There is no pre-image for 1.This contradicts the fact that range is {1, 2, 3}.Case-3: Let f(z)≠2 be true.Then, f(x)=1 and f(y)≠1 are false.So,  f(x)≠1 and f(y)=1⇒f(x)=2, f(y)=1 and f(z)=3⇒f(y)=1⇒f-1(1)=y
So, the answer is (b).

Page No 2.73:

Question 14:

Which of the following functions form Z to itself are bijections?
(a) f(x)=x3
(b) f(x)=x+2
(c) f(x)=2x+1
(d) f(x)=x2+x

ANSWER:

(af is not onto because for = 3∈Co-domain(Z), there is no value of x∈Domain(Z)x3=3⇒x=3√3∉Zf is not onto.So, fis not a bijection.

(b) Injectivity:
Let x and y be two elements of the domain (Z), such that
 x+2=y+2⇒x=y
So, f is one-one.

Surjectivity:
Let y be an element in the co-domain (Z), such that
y=f(x)⇒y=x+2⇒x=y-2∈Z (Domain)
f is onto.
So, f is a bijection.

(cf(x)=2x+1 is not onto because if we take 4 ∈ Z(co domain), then 4=f(x)⇒4=2x+1⇒2x=3⇒x=32∉ZSo, f  is not a bijection.(df(0)=02+0=0and f(-1)=(-1)2+(-1)=1-1=0⇒0 and -1 have the same image.⇒f is not one-one.So, f is not a bijection.

So, the answer is (b).

Page No 2.73:

Question 15:

Which of the following functions from A={x : -1≤x≤1} to itself are bijections?
(a) f(x)=x2
(b) g(x)=sin(π x2)
(c) h(x)=|x|
(d) k(x)=x2

ANSWER:

(a) Range of f =[-12, 12] ≠ ASo, f is not a bijection.(b) Range =[sin(-π2), sin(π2)]=[-1,1]=ASo, g is a bijection.(ch(-1)=|-1|=1and h(1)=|1|=1⇒-1 and 1 have the same imagesSo, h is not a bijection.(dk(-1)=(-1)2=1and k(1)=(1)2=1⇒-1 and 1 have the same imagesSo, k is not a bijection.
So, the answer is (b).

Page No 2.73:

Question 16:

Let A={x : -1≤x≤1} and f : AA such that f(x)=x|x|, then f is
(a) a bijection
(b) injective but not surjective
(c) surjective but not injective
(d) neither injective nor surjective

ANSWER:

Injectivity:
Let x and y be any two elements in the domain A.

Case-1: Let x and y be two positive numbers, such that
f(x)=f(y)⇒x|x|=y|y|⇒x(x)=y(y)⇒x2=y2⇒x=y

Case-2: Let x and y be two negative numbers, such that
f(x)=f(y)⇒x|x|=y|y|⇒x(-x)=y(-y)⇒-x2=-y2⇒x2=y2⇒x=y

Case-3: Let be positive and y be negative.
Then, xyf(x)=x|x| is positive and f(y)=y|y| is negative⇒f(x)≠f(y)So, x≠yf(x)≠f(y)
So, f is one-one.

Surjectivity:
Let y be an element in the co-domain, such that y = f (x)
Case-1: Let y>0. Then, 0<y≤1y=f(x)=x|x|>0⇒x>0⇒|x|=xf(x)=yx|x|=yx(x)=yx2=yx=√y ∈ A (We do not get ±, as x>0)Case-2: Let y<0Then-1≤y<0y=f(x)=x|x|<0⇒x<0⇒|x|=-xf(x)=yx|x|=yx(-x)=y⇒-x2=yx2=-yx=-√-y ∈ A (We do not get ±, as x>0)
is onto
is a bijection.
So, the answer is (a).

Page No 2.73:

Question 17:

If the function f : R→A given by f(x)=x2x2+1 is a surjection, then A =
(a) R
(b) [0, 1]
(c) [0, 1)
(d) [0, 1)

ANSWER:


As f is surjective, range of f=co-domain of fA= range of f ∵f(x)= x2x2+1,  y=x2x2+1⇒y(x2+1)= x2⇒(y-1)x2+y= 0⇒x2= –y(y-1)⇒x=√y(1-y)⇒y(1-y)≥0⇒y∈[0, 1)⇒Range of f= [0, 1)⇒A= [0, 1)
So, the answer is (d).

Page No 2.73:

Question 18:

If a function f : [2, ∞) → B defined by f(x)=x2-4x+5 is a bijection, then B =
(a) R
(b) [1, ∞)
(c) [4, ∞)
(d) [5, ∞)

ANSWER:

Since f is a bijection, co-domain of f = range of f
B = range of f
Given: f(x)=x2-4x+5Let f(x)=yy=x2-4x+5⇒x2-4x+(5-y)=0∵Discrimant, D=b2-4ac≥0,(-4)2-4×1×(5-y)≥0⇒16-20+4y≥0⇒4y≥4⇒y≥1⇒y∈[1, ∞)⇒Range of f=[1, ∞)⇒B=[1, ∞)
So, the answer is (b).

Page No 2.74:

Question 19:

The function f : RR defined by
f(x)=(x-1) (x-2) (x-3) is
(a) one-one but not onto
(b) onto but not one-one
(c) both one and onto
(d) neither one-one nor onto

ANSWER:

f(x)=(x-1)(x-2)(x-3)

Injectivity:
f(1)=(1-1)(1-2)(1-3)=0f(2)=(2-1)(2-2)(2-3)=0f(3)=(3-1)(3-2)(3-3)=0⇒ f(1)=f(2)=f(3)=0
So, is not one-one.

Surjectivity:
Let y be an element in the co domain R, such that
y=f(x)⇒y=(x-1)(x-2)(x-3)Since yR and xRf is onto.

So, the answer is (b).

Page No 2.74:

Question 20:

The function f : [-1/2, 1/2, 1/2]→[-π/2, π/2] , defined by f(x)=sin-1 (3x-4x3), is
(a) bijection
(b) injection but not a surjection
(c) surjection but not an injection
(d) neither an injection nor a surjection

ANSWER:

f(x)=sin-1(3x-4x3)⇒f(x)=3sin-1x

Injectivity:
Let x and be two elements in the domain [-12, 12] , such that
f(x)=f(y)⇒3sin-1x=3sin-1y⇒sin-1x=sin-1yx=y
So, f is one-one.

Surjectivity:
Let y be any element in the co-domain, such that
f(x)=y⇒3sin-1(x)=y⇒sin-1(x)=y3⇒x=siny3∈[-12, 12]
is onto.
f is a bijection.
So, the answer is (a).

Page No 2.74:

Question 21:

Let f : RR be a function defined by f(x)=e|x|-exex+ex. Then,
(a) f is a bijection
(b) f is an injection only
(c) f is surjection on only
(d) f is neither an injection nor a surjection

ANSWER:

(d) f is neither an injection nor a surjection

f : RR

f(x)=e|x|-exex+exFor x=-2 and -3∈ R f(-2) =e|-2|-e2e-2+e2           =e2-e2e-2+e2           = 0& f(-3) =e|-3|-e3e-3+e3              =e3-e3e-3+e3              = 0Hence, for different values of x we are getting same values of f(x)That means , the given function is many one .
Therefore, this function is not injective.

For x<0f(x) =0For x>0f(x) =exexex+ex        =ex+exex+ex-2exex+ex        =1-2exex+exThe value of 2exex+ex is always  positive.Therefore, the value of f(x) is always less than 1Numbers more than 1 are not included in the range but they are included in codomain.As the codomain is R.∴ Codomain≠RangeHence, the given function is not onto .
Therefore, this function is not surjective .

Page No 2.74:

Question 22:

Let f : R-{n}→R be a function defined by

f(x)=xmxn, where mn. Then,

(a) f is one-one onto
(b) f is one-one into
(c) f is many one onto
(d) f is many one into

ANSWER:

Injectivity:
Let x and be two elements in the domain R-{n}, such that
f(x)=f(y)⇒xmxn=ymyn⇒(xm)(yn)=(xn)(ym)⇒xynxmy+mn=xymxny+mn⇒(mn)x=(mn)yx=y
So, f is one-one.

Surjectivity:
Let y be an element in the co domain R, such that
f(x)=yxmxn=yxm=xynynym=xyxnym=x(y-1)⇒x=nymy-1, which is not defined for y =1So, 1 ∈ R (co domain) has no pre image in R-{n}
f is not onto.
Thus, the answer is (b).

Page No 2.74:

Question 23:

Let f : RR be a function defined by f(x)=x2-8x2+2. Then,  f is
(a) one-one but not onto
(b) one-one and onto
(c) onto but not one-one
(d) neither one-one nor onto

ANSWER:

Injectivity:
Let x and y be two elements in the domain (R), such that
f(x)=f(y)⇒x2-8x2+2=y2-8y2+2⇒(x2-8)(y2+2)=(x2+2)(y2-8)⇒x2y2+2x2-8y2-16=x2y2-8x2+2y2-16⇒10x2=10y2⇒x2=y2⇒xy
So, f is not one-one.

Surjectivity:
f(-1)=(-1)2-8(-1)2+2=1-81+2=-73 and f(1)=(1)2-8(1)2+2=1-81+2=-73⇒f(-1)=f(1)=-73
f is not onto.
The correct answer is (d).

Page No 2.74:

Question 24:

f : RR is defined by f(x)=ex2-ex2ex2+ex2 is
(a) one-one but not onto
(b) many-one but onto
(c) one-one and onto
(d) neither one-one nor onto

ANSWER:

(d) neither one-one nor onto

We have,f(x)=ex2-ex2ex2+ex2Here, -2, 2∈RNow, 2≠-2But, f(2)=f(-2)Therefore, function is not one-one.And,The minimum value of the function is 0 and maximum value is 1That is range of the function is [0, 1] but the co-domain of the function is given R.Therefore, function is not onto.∴function is neither one-one nor onto.

Page No 2.74:

Question 25:

The function f : RRf(x)=x2 is
(a) injective but not surjective
(b) surjective but not injective
(c) injective as well as surjective
(d) neither injective nor surjective

ANSWER:

Injectivity:
Let x and y be any two elements in the domain (R), such that f(x) = f(y). Then,

x2=y2⇒x=±y


So, f is not one-one.

Surjectivity:
As f(-1)=(-1)2=1and f(1)=12=1, f(-1)=f(1)
So, both -1 and 1 have the same images.
f is not onto.
So, the answer is (d).

x2+x+1=y2+y+1(x2−y2)+(x−y)=0(x+y)(x−y)+(x−y)=0(x−y)(x+y+1)=0x−y=0 (x+y+1) cannot be zero because x and y are natural numbers)x=y

Page No 2.74:

Question 26:

A function f from the set of natural numbers to the set of integers defined by

f(n){n-12,when n is odd-n2,when n is evenis

(a) neither one-one nor onto
(b) one-one but not onto
(c) onto but not one-one
(d) one-one and onto

ANSWER:

Injectivity:
Let x and y be any two elements in the domain (N).
Case-1: Both x and y are even.Let f(x)=f(y)⇒-x2=-y2⇒-x=-yx=yCase-2: Both x and y are odd.Let f(x)=f(y)⇒x-12=y-12⇒x-1=y-1⇒x=yCase-3: Let x be even and y be odd. Then, f(x)=-x2and f(y)=y-12Then, clearly xy ⇒f(x)≠f(y)From all the cases, f is one-one.

Surjectivity:
Co-domain of f=Z={…,-3, -2, -1, 0, 1, 2, 3, ….}Range of f={…, -3-12, -(-2)2,-1-12, 02, 1-12, -22, 3-12, …}⇒Range of f={…,-2, 1, -1, 0, 0, -1, 1,…}⇒Range of f={…, -2, -1, 0, 1, 2, ….}⇒Co-domain of f=Range of f
⇒ f is onto.
So, the answer is (d).

Page No 2.74:

Question 27:

Which of the following functions from A={x ∈ R : -1≤x≤1} to itself are bijections?

(a) f(x)=|x|
(b) f(x)=sinπ x2
(c) f(x)=sinπ x4
(d) None of these

ANSWER:

(b) f(x)=sinπ x2

It is clear that  f(x) is one-one.

Range of f=[sinπ(-1)2, sinπ(1)2]=[sin -π2, sinπ2]=[-1,1]= A=Co domain of f
⇒ f is onto.
So, f is a bijection.

Page No 2.74:

Question 28:

Let f : ZZ be given by f(x)={x2, if x is even0, if x is odd
Then,  f is
(a) onto but not one-one
(b) one-one but not onto
(c) one-one and onto
(d) neither one-one nor onto

ANSWER:

Injectivity:
Let x and y be two elements in the domain (Z), such that
f(x)=f(y)Case-1: Let both x and y be even.Then,f(x)=f(y)⇒x2=y2⇒x=yCase-2: Let both x and y be odd.Then,f(x)=f(y)⇒0=0Here, we cannot determine whether x=y.
So, f is not one-one.

Surjectivity:
Let y be an element in the co-domain (Z), such that
Co-domain of f =Z={0, ±1, ±2, ±3, ±4, …}Range of f={0, 0, ±22, 0, ±42 ,…}={0, ±1, ±2, …}⇒Co-domain of f=Range of f
⇒ f is onto.
So, the answer is (a).

Page No 2.74:

Question 29:

The function f : RR defined by f(x)=6x+6|x| is
(a) one-one and onto
(b) many one and onto
(c) one-one and into
(d) many one and into

ANSWER:

(d) many one and into

Graph of the given function is as follows :



A line parallel to X axis is cutting the graph at two different values.

Therefore, for two different values of x we are getting the same value of y .
That means it is many one function .

From the given graph we can see that the range is [2,∞) 
and R is the codomain of the given function .
Hence, Codomain≠Range
Therefore, the given function is into .
 

Page No 2.75:

Question 30:

Let f(x)=x2 and g(x)=2x. Then, the solution set of the equation fog (x)=gof (x) is
(a) R
(b) {0}
(c) {0, 2}
(d) none of these

ANSWER:


Since (fog)(x)=(gof)(x), f(g(x))=g(f(x))⇒f(2x)=g(x2)⇒(2x)2=2x2⇒22x=2x2⇒x2=2xx2-2x=0⇒x(x-2)=0⇒x=0, 2⇒x∈{0, 2}
So, the answer is (c).

Page No 2.75:

Question 31:

If f : RR is given by f(x)=3x-5, then f-1(x)
(a) is given by 13x-5
(b) is given by x+53
(c) does not exist because f is not one-one
(d) does not exist because f is not onto

ANSWER:

Clearly, f is a bijection.
So, f -1 exists.
Let f-1(x)=y                   …(1)⇒f(y)=x⇒3y-5=x⇒3y=x+5⇒y=x+53⇒f-1(x)=x+53          [from (1)]
So, the answer is (b).

Page No 2.75:

Question 32:

If g (f (x))=|sin x| and f (g (x))=(sin √x)2, then
(a) f(x)=sin2 xg(x)=√x
(b) f(x)=sin xg(x)=|x|
(c) f(x)=x2, g(x)=sin √x
(d) f and g cannot be determined.

ANSWER:

If we solve it  by the trial-and-error method, we can see that (a) satisfies the given condition.
From (a):
f(x)=sin2x and g(x)=√xf(g(x))=f(√x)=sin2 √x=(sin √x)2
So, the answer is (a).

Page No 2.75:

Question 33:

The inverse of the function f : R→{x ∈ R : x < 1} given by f(x)=exexex+ex is
(a) 12 log 1+x1-x

(b) 12 log 2+x2-x

(c) 12 log 1-x1+x

(d) none of these

ANSWER:

Let f-1(x)=y         …(1)⇒f(y)=xeyeyey+ey=xey(e2y-1)ey(e2y+1)=x⇒(e2y-1)=x(e2y+1)⇒e2y-1=xe2y+xe2y(1-x)=x+1⇒e2y=1+x1-x⇒2y=loge (1+x1-x)⇒y=12loge (1+x1-x)⇒f-1(x)=12loge (1+x1-x)                [from (1)]
So, the answer is (a).

Page No 2.75:

Question 34:

Let A={x ∈ R : x ≥ 1}. The inverse of the function, f : AA given by f(x)=2x (x-1), is

(a) (12)x (x-1)

(b) 12 {1+√1+4 log2 x}

(c) 12 {1-√1+4 log2 x}

(d) not defined

ANSWER:

Let f-1(x)=y…(1)⇒f(y)=x⇒2y(y-1)=x⇒2y2-y=xy2-y=log2 xy2-y+14=log2 x+14⇒(y-12)2=4log2 x+14⇒y-12=±√4log2 x+12⇒y=12±√4log2 x+12⇒y=12+√4log2 x+12                      (∵ y ≥1)So, f-1(x)=12(1+√1+4log2 x )       [from (1)]
So, the answer is (b).

Page No 2.75:

Question 35:

Let A={x ∈ R : x ≤1} and f : AA  be defined as f(x)=x (2-x). Then, f-1 (x) is
(a) 1+√1-x
(b) 1-√1-x
(c) √1-x
(d) 1±√1-x

ANSWER:

Let y be the element in the codomain R such that        f-1(x)=y …(1)⇒f(y)=x and y ≤1⇒y(2-y)=x⇒2yy2=xy2-2y+x=0⇒y2-2y=-xy2-2y+1=1-x⇒(y-1)2=1-xy-1=±√1-xy=1±√1-xy=1-√1-x        (∵ y ≤1)
The correct answer is (b).

Page No 2.75:

Question 36:

Let f(x)=11-x. Then, {f o (fof)} (x)
(a) x for all x ∈ R
(b) x for all x ∈ R-{1}
(c) x for all x ∈ R-{0, 1}
(d) none of these

ANSWER:

Domain of f:1-x≠0⇒x≠1Domain of f=R-{1}Range of f:y=11-x⇒1-x=1yx=1-1yx=y-1yy≠0Range of f=R-{0}So, f:R-{1}→R-{0} and f:R-{1}→R-{0} Range of f is not a subset of the domain of f.Domain (fof)={xx∈domain of f and f(x)∈domain of f}Domain (fof)={xxR-{1} and 11-xR-{1}} Domain (fof)={xx ≠1 and 11-x≠1}Domain (fof)={xx ≠1 and 1-x≠1}Domain (fof)={xx ≠1 and x≠0}Domain (fof)=R-{0, 1}(fof)(x)=f(f(x))=f(11-x)=11-11-x=1-x1-x-1=1-xx=x-1xFor range of fofx≠0Now, fof:R-{0, 1}→R -{0} and f:R-{1}→R-{0}Range of fof is not a subset of domain of f.Domain(f o (fof))={xx∈domain of fof and (fof)(x)∈domain of f}Domain (f o (fof))={xxR-{0, 1} and x-1xR-{1}} Domain(f o (fof))={xx ≠0, 1 and x-1x≠1}Domain (f o (fof))={xx ≠0, 1 and x-1≠x}Domain (f o (fof))={xx ≠0, 1 and xR}Domain (f o (fof))=R-{0, 1}(fo(fof))(x)=f((fof)(x))=f(x-1x)=11-x-1x=xxx+1=xSo, (fo(fof))(x)=x, where x≠0,1
So, the answer is (c).

Page No 2.75:

Question 37:

If the function f : RR be such that f(x)=x-[x], where [x] denotes the greatest integer less than or equal to x, then f-1 (x) is

(a) 1x-[x]

(b) [x] − x

(c) not defined

(d) none of these

ANSWER:

f(x) = x – [x]
We know that the range of f is [0, 1).
Co-domain of f = R
As range of f ≠Co-domain of f,  f is not onto.
⇒ f is not a bijective function.
So,  f -1 does not exist.
Thus, the answer is (c).

Page No 2.75:

Question 38:

If F : [1, ∞)→[2, ∞) is given by f(x)=x+1x, then f-1 (x) equals

(a) x+√x2-42

(b) x1+x2

(c) x-√x2-42

(d) 1+√x2-4

ANSWER:

Let f-1(x) = yf(y)=xy+1y=xy2+1=xyy2-xy+1=0⇒y2-2×y×x2+(x2)2-(x2)2+1=0⇒y2-2×y×x2+(x2)2=x2-14⇒(yx2)2=x2-14⇒yx2=√x2-42⇒y=x2+√x2-42⇒y=x+√x2-42⇒f-1(x)=x+√x2-42

So, the answer is (a).

Page No 2.75:

Question 39:

Let g(x)=1+x-[x] and f(x)={-1,x<00,x=0, 1,x>0, where [x] denotes the greatest integer less than or equal to x. Then for all xf (g (x)) is equal to
(a) x
(b) 1
(c) f(x)
(d) g(x)

ANSWER:

(b) 1

When, -1<x<0Then, g(x)=1+x-[x]               =1+x-(-1)=2+xf(g(x))=1 When, x=0Then, g(x)=1+x-[x]               =1+x-0=1+xf(g(x))=1When, x>1Then, g(x)=1+x-[x]               =1+x-1=xf(g(x))=1

Therefore, for each interval f(g(x))=1

Page No 2.75:

Question 40:

Let f(x)=α xx+1, x≠-1. Then, for what value of α is f (f(x))=x?
(a) √2
(b) -√2
(c) 1
(d) −1

ANSWER:

(d) −1

  f(f(x))=x⇒ f(αxx+1)=xα(αxx+1)(αxx+1)+1=xα2xαx+x+1=xα2x=αx2+x2+xα2xαx2-x2-x=0⇒α2xαx2-(x2+x)=0Solving for the α we get,α=-(-x2)±√(-x2)2-4×x×[-(x2+x)]2x  =x2±√x4+4x3+4x22x  =x+1, -1Here, -1 is independent of x,∴for, α=-1, f(f(x))=x   

Page No 2.76:

Question 41:

The distinct linear functions that map [−1, 1] onto [0, 2] are
(a) f(x)=x+1, g(x)=-x+1
(b) f(x)=x-1, g(x)=x+1
(c) f(x)=-x-1, g(x)=x-1
(d) None of these

ANSWER:

Let us substitute the end-points of the intervals in the given functions. Here, domain = [-1, 1] and range =[0, 2]
By substituting -1 or 1 in each option, we get:

Option (a):
f(-1)=-1+1=0 and f(1)=1+1=2g(-1)=1+1=2 and g(1)=-1+1=0
So, option (a) is correct.

Option (b):
f(-1)=-1-1=-2 and f(1)=1-1=0g(-1)=-1+1=0 and g(1)=1+1=2
Here, f(-1) gives -2∉[0, 2]
So, (b) is not correct.

Similarly, we can see that (c) is also not correct.

Page No 2.76:

Question 42:

Let f : [2, ∞)→X be defined by f(x)=4xx2. Then, f is invertible if X =
(a) [2, ∞)
(b) (-∞, 2]
(c) (-∞, 4]
(d) [4, ∞)

ANSWER:

Since f is invertible, range of f = co domain of f = X
So, we need to find the range of f to find X.
For finding the range, let
f(x)=y⇒4xx2=yx2-4x=-yx2-4x+4=4-y⇒(x-2)2=4-yx-2=±√4-yx=2±√4-yThis is defined only when 4-y≥0⇒y≤4X=Range of f=(-∞,4]
So, the answer is (c).

Page No 2.76:

Question 43:

If f : R→ (-1, 1) is defined by f(x)=-x|x|1+x2, then f-1 (x) equals
(a) √|x|1-|x|

(b) Sgn (x) √|x|1-|x|

(c) -√x1-x

(d) None of these

ANSWER:

(b) -Sgn (x) √|x|1-|x|

We have, f(x)=-x|x|1+x2    x∈(-1, 1)Case-(I)When, x<0,Then, |x|=-xAnd f(x)>0Now,f(x)=-x(-x)1+x2⇒y=x21+x2⇒y1=x21+x2⇒y+1y-1=x2+1+x2x2-1-x2           [Using Componendo and dividendo]⇒y+1y-1=2x2+1-1⇒-y+1y-1=2x2+1⇒2y1-y=2x2⇒y1-y=x2⇒x=-√y1-y                         (As x<0)⇒x=-√|y|1-|y|                        [As y>0]To find the inverse interchanging x and y we get,f-1(x)=-√|x|1-|x|            …(i)Case-(II)When, x>0,Then, |x|=xAnd f(x)<0Now,f(x)=-x(x)1+x2⇒y=-x21+x2⇒y1=-x21+x2⇒y+1y-1=-x2+1+x2-x2-1-x2           [Using Componendo and dividendo]⇒y+1y-1=1-2x2-1⇒1+y1-y=12x2+1⇒1-y1+y=2x2+1⇒-2y1+y=2x2⇒x2=-y1+yx=√-y1+y                         (As x>0)⇒x=√|y|1-|y|                        [As y<0]To find the inverse interchanging x and y we get,f-1(x)=√|x|1-|x|            …(ii)Case-(III)When, x=0,Then, f(x)=0Hence, f-1(x)=0     …(iii)Combinig equation (i) , (ii) and (iii) we get,f-1(x)=-Sgn(x)√|x|1-|x|  

Page No 2.76:

Question 44:

Let [x] denote the greatest integer less than or equal to x. If f(x)=sin-1xg(x)=[x2] and h(x)=2x,12≤x≤1√2, then
(a) fogoh(x)=π2
(b) fogoh(x)=π
(c) hofog=hogof
(d) hofoghogof

ANSWER:

(c) hofog=hogof

We have,g(x)=[x2]       =0          ( As 12≤x≤ 1√2∴ 14≤x2≤ 12)fog(x)=f(g(x))=sin-1(0)                       =0hofog(x)=h(f(g(x)))=2×0=0

Andf(x)=sin-1xNow,forx∈[12, 1√2]f(x)∈[π6, π4]f(x)∈[0.52, 0.78]gof(x)=0    (As, f(x)∈[0.52, 0.78])                       =0hogof(x)=h(g(f(x)))=2×0=0

 ∴ hofog=hogof=0
 

Page No 2.76:

Question 45:

If g(x)=x2+x-2 and 12 gof(x)=2x2-5x+2, then f(x) is equal to
(a) 2 x-3
(b) 2 x+3
(c) 2 x2+3x+1
(d) 2 x2-3x-1

ANSWER:

We will solve this problem by the trial-and-error method.
Let us check option (a) first.
If f(x) = 2x-312(gof)(x) = g(f(x))= 12g(2x-3)= 12[(2x-3)2+(2x-3)-2]= 12[4x2+9-12x+2x-3-2]= 12[4x2-10x+4]= 2x2-5x+2
The given condition is satisfied by (a).
So, the answer is (a).

Page No 2.76:

Question 46:

If f(x)=sin2 x and the composite function g(f(x))=|sin x|, then g(x) is equal to
(a) √x-1
(b) √x
(c) √x+1
(d) -√x

ANSWER:

(b)

 If we take g(x) = √x, theng(f(x)) = g(sin2x) = √sin2x = ±sin x = |sin x|

So, the answer is (b).

Page No 2.76:

Question 47:

If f : RR is given by f(x)=x3+3, then f-1(x) is equal to
(a) x1/3-3
(b) x1/3+3
(c) (x-3)1/3
(d) x+31/3

ANSWER:

(c)

Let f-1(x) = yf(y) = xy3+3 = xy3 = x-3⇒y = 3√x-3 ⇒y = (x-3)13
So, the answer is (c).

Page No 2.76:

Question 48:

Let f(x)=x3 be a function with domain {0, 1, 2, 3}. Then domain of f-1 is
(a) {3, 2, 1, 0}
(b) {0, −1, −2, −3}
(c) {0, 1, 8, 27}
(d) {0, −1, −8, −27}

ANSWER:

(c) {0, 1, 8, 27}

f(x)=x3Domain = {0, 1, 2, 3}Range = {03, 13, 23, 33} = {0, 1, 8, 27}So, f = {(0, 0), (1, 1), (2, 8), (3, 27)}f-1 = {(0, 0), (1, 1), (8, 2), (27, 3)}Domain of f-1 = {0, 1, 8, 27}

Page No 2.76:

Question 49:

Let f : RR be given by f(x)=x2-3. Then, f-1 is given by
(a) √x+3
(b) √x+3
(c) x+√3
(d) None of these

ANSWER:

(d)

Let f-1(x)=yf(y)=xy2-3=xy2=x+3y=±√x+3
So, the answer is (d).

Page No 2.76:

Question 50:

Mark the correct alternative in the following question:

Let f : R → R be given by f(x) = tanx. Then, f -1(1) is

(a) π4                              (b) {nπ+π4:nZ}                              (c) does not exist                              (d) none of these

ANSWER:

We have,f:RR is given byf(x)=tanxf-1(x)=tan-1x∴ f-1(1)=tan-11={nπ+π4:nZ}

Hence, the correct alternative is option (b).

Page No 2.76:

Question 51:

Mark the correct alternative in the following question:

Let f : R → R be defined as f(x) = {2x, if x>3     x2, if 1<x≤33x, if x≤1      

Then, find f(-1) + f(2) + f(4)

(a) 9                              (b) 14                              (c) 5                              (d) none of these

ANSWER:

We have,f(x)={2x, if x>3     x2, if 1<x≤33x, if x≤1      Now,f(-1)+f(2)+f(4)=3(-1)+22+2(4)=-3+4+8=9

Hence, the correct alternative is option (a).

Page No 2.76:

Question 52:

Mark the correct alternative in the following question:

Let A = {1, 2, … , n} and B = {ab}. Then the number of subjections from A into B is

(a) nP2                               (b) 2n – 2                               (c) 2n – 1                               (d) nC2

ANSWER:

As, the number of surjections from A to B is equal to the number of functions from A to B minus the number of functions from A to B whose images are proper subsets of B.

And, the number of functions from a set with n number of elements into a set with m number of elements = mn

So, the number of subjections from A into B where A = {1, 2, … , n} and B = {ab} is 2n – 2.           (As, two functions can be many-one into functions)

Hence, the correct alternative is option (b).

Page No 2.76:

Question 53:

Mark the correct alternative in the following question:

If the set A contains 5 elements and the set B contains 6 elements, then the number of one-one and onto mappings from A to B is

(a) 720                                                 (b) 120                                                 (c) 0                                                 (d) none of these

ANSWER:

As, the number of bijection from A into B can only be possible when provided n(A)≥n(B)But here n(A)<n(B)So, the number of bijection i.e. one-one and onto mappings from A to B=0

Hence, the correct alternative is option (c).

Page No 2.76:

Question 54:

Mark the correct alternative in the following question:

If the set A contains 7 elements and the set B contains 10 elements, then the number one-one functions from A to B is

(a) 10C7                                            (b) 10C7 × 7!                                              (c) 710                                              (d)107

ANSWER:

As, the number of one-one functions from A to B with m and n elements, respectively = nPm = nCm × m!

So, the number of one-one functions from A to B with 7 and 10 elements, respectively = 10P7 = 10C7 × 7!

Hence, the correct alternative is option (b).

Page No 2.76:

Question 55:

Mark the correct alternative in the following question:

Let f : R – {35} → R be defined by f(x) = 3x+25x-3. Then,

(a) f -l(x) = f(x)                         (b) f -1(x) = –f(x)                         (c) fof(x) = –x                         (d) -1(x) = 119f(x)

ANSWER:

We have,

f : R – {35} → R is defined by f(x) = 3x+25x-3

fof(x)=f(f(x))=f(3x+25x-3)=3(3x+25x-3)+25(3x+25x-3)-3=(9x+65x-3)+2(15x+105x-3)-3=(9x+6+10x-65x-3)(15x+10-15x+95x-3)=19x19=xLet y=3x+25x-3⇒5xy-3y=3x+2⇒5xy-3x=3y+2⇒x(5y-3)=3y+2⇒x=3y+25y-3⇒f-1(y)=3y+25y-3So, f-1(x)=3x+25x-3=f(x)

Hence, the correct alternative is option (a).

Page No 2.77:

Question 56:

Let f : R → R be defined by f(x)=1x. Then,  f is
(a) one-one
(b) onto
(e) bijective
(d) not defined

ANSWER:

Given: The function f : R → R be defined by f(x)=1x.

To check f is one-one:

Let f(x1)=f(x2)⇒1x1=1x2⇒x1=x2Hence, f is one-one.To check f is onto: Since, y=1xx=1yyR-{0}≠RThere is no pre-image of y=0.Hence, f is not onto.

Hence, the correct option is (a).

Page No 2.77:

Question 57:

Let f : R → R be defined by f(x) = 3x2 – 5 and g : R → R by g(x)=xx2+1. Then (gof) (x) is
(a) 3x2−59x4−30x2+26

(b) 3x2−59x4−6x2+26

(c) 3x2x4+2x2−4

(d) 3x29x4+30x2−2

ANSWER:

Given: f(x) = 3x2 – 5 and g(x)=xx2+1

(gof)(x)=g(f(x))            =g(3x2-5)            =3x2-5(3x2-5)2+1            =3x2-5(3x2)2+52-2(3x2)(5)+1            =3x2-59x4+25-30x2+1            =3x2-59x4-30x2+26

​Hence, the correct option is (a).

Page No 2.77:

Question 58:

Which of the following functions from Z to Z are bijections?
(a) f(x) = x3
(b) f(x) = x + 2
(c) f(x) = 2x + 1
(d) f(x) = x2 + 1

ANSWER:

Given: f : Z → Z

(a) f(x) = x3

It is one-one but not onto.

Thus, it is not bijective.

(b) f(x) = x + 2

It is one-one and onto.

Thus, it is bijective.

(c) f(x) = 2x + 1

It is one-one but not onto.

Thus, it is not bijective.

(d) f(x) = x2 + 1

It is neither one-one nor onto.

Thus, it is not bijective.

Hence, the correct option is (b).

Page No 2.77:

Question 59:

Let f : A → B and g : B → C be the bijective functions. Then, (gof)–1 =
(a) f–1o g–1
(b) fog
(c) g–1of–1
(d) gof

ANSWER:

Given: f : A → B and g : B → C be the bijective functions

Since, f:ABThus, f-1:BA       …(1)Since, g:BCThus, g-1:CB       …(2)From (1) and (2), we getf-1og-1:CA          …(3)Also, gof:AC⇒(gof)-1:CA       …(4)Therefore, (gof)-1=f-1og-1

​Hence, the correct option is (a).

Page No 2.77:

Question 60:

Let f : N → R be the function defined by f(x)=2x−12 and g : Q → R be another function defined by g(x) = x + 2. Then, (gof) (3/2) is
(a) 1

(b) 2

 (c) 72

(d) none of these

ANSWER:

Given: f(x)=2x−12 and g(x) = x + 2

(gof)(x)=g(f(x))            =g(2x-12)            =2x-12+2            =2x-1+42            =2x+32(gof)(32)=2(32)+32              =3+32              =62              =3

​Hence, the correct option is (d).

Page No 2.77:

Question 1:

The total number of functions from the set A = (1, 2, 3, 4 to the set B = abc) is _________.

ANSWER:

Given: f:AB where A={1, 2, 3, 4} and B={abc}

Number of elements in = 4
Number of elements in = 3
Each element of A have 3 options to form an image.

Thus, Number of functions that can be formed = 3 × 3 × 3 × 3 = 81

​Hence, the total number of functions from the set A = {1, 2, 3, 4} to the set B = {abc} is 81.

Page No 2.77:

Question 2:

The total number of one-one functions from the set A = {a, bc} to the set B = {xyzt} is _________.

ANSWER:

Given: f:AB where A={abc} and B={xyzt}

Number of elements in = 3
Number of elements in = 4

To form a one-one function,

Element a ∈ A have 4 options to form an image.
Element b ∈ A have 3 options to form an image.
Element c ∈ A have 2 options to form an image.

Thus, Number of one-one functions that can be formed = 4 × 3 × 2 = 24

​Hence, the total number of one-one functions from the set A = {a, bc} to the set B = {xyzt} is 24.

Page No 2.77:

Question 3:

The total number of onto functions from the set A = (1, 2, 3, 4, 5) to the set B = {xy} is _________.

ANSWER:

Given: f:AB where A={1, 2, 3, 4, 5} and B={xy}

Number of elements in = 5
Number of elements in = 2


Each Element of A have 2 options to form an image.

Thus, Total number of functions that can be formed = 2 × 2 × 2 × 2 × 2 = 32

Number of functions having only one image i.e., {x} = 1
Number of functions having only one image i.e., {y} = 1

Thus, Number of onto functions that can be formed = 32 − 1 − 1 = 30

​Hence, the total number of onto functions from the set A = {1, 2, 3, 4, 5} to the set B = {xy} is 30.

Page No 2.77:

Question 4:

The domain of the real function f(x)=√16−x2 is _________.

ANSWER:

Given: f(x)=√16−x2

To find the domain, we find the real values of x for which the function is defined.

16-x2≥0⇒16≥x2⇒x2≤16⇒x≤4 and x≥-4⇒-4≤x≤4⇒x∈[-4, 4]

Hence, the domain of the real function f(x)=√16−x2 is [−4, 4].

Page No 2.77:

Question 5:

The domain of the real function f(x)=x√9−x2 is ___________.

ANSWER:

Given: f(x)=x√9−x

To find the domain, we find the real values of x for which the function is defined.

xR and 9-x2>0⇒xR and 9>x2⇒xR and x2<9⇒xR and -3<x<3⇒-3<x<3⇒x∈(-3, 3)

Hence, the domain of the real function f(x)=x√9−x2 is  (−3, 3).

Page No 2.77:

Question 6:

The range of the function f : R → R given by f(x)=x+√x2 is  _________.

ANSWER:

Given: f(x)=x+√x

f(x)=x+√x2      =x+|x|      ={x+x         ,x≥0xx         ,x<0      ={2x         ,x≥00           ,x<0

To find the range, we find the real values of y obtained.

y=2x when x≥0⇒x=y2≥0⇒y≥0⇒y∈[0, ∞)         …(1)y=0 when x<0    …(2)Thus, from (1) and (2),y∈[0, ∞)

Hence, the range of the function f : R → R given by f(x)=x+√x2 is [0, ∞).

Page No 2.77:

Question 7:

The range of the function f : R –{–2) → R given by f(x)=x+2|x+2| is _________.

ANSWER:

Given: f(x)=x+2|x+2|

f(x)=x+2|x+2|      ={x+2x+2                , x+2≥0x+2-(x+2)          , x+2<0      ={1                , x+2≥0-1             , x+2<0

To find the range, we find the real values of y obtained.

Hence, the range of the function f : R –{–2) → R given by f(x)=x+2|x+2| is {–1, 1}.

Page No 2.77:

Question 8:

If f : C → C is defined by f(x) = 8x3, then f–1(8) = . _________.

ANSWER:

Given: f(x) = 8x3

f(x)=8x3⇒y=8x3⇒x3=y8⇒x=(y8)13Thus, f-1(x)=(x8)13f-1(8)=(88)13         =113         =1, ωω2       (∵ f:CC)where, ω is the cube root of unity.

Hence, if f : C → C is defined by f(x) = 8x3, then f1(8) = 1, ωω2.

Page No 2.77:

Question 9:

If f : R → R is defined by f(x) = 8x3 then, f–1(8) = _________.

ANSWER:

Given: f(x) = 8x3

f(x)=8x3⇒y=8x3⇒x3=y8⇒x=(y8)13Thus, f-1(x)=(x8)13f-1(8)=(88)13         =113         =1                 (∵ f:RR)

Hence, if f : R → R is defined by f(x) = 8x3 then f1(8) = 1.

Page No 2.78:

Question 10:

If f : R – {0} → R – {0} is defined as f(x)=23x, then f–1(x) = ___________.

ANSWER:

Given: A function f : R – {0} → R – {0} is defined as f(x)=23x

f(x)=23xy=23x⇒3x=2yx=23yThus, f-1(x)=23x

Hence, if f : R – {0} → R – {0} is defined as f(x)=23x, then f1(x) = 23x.

Page No 2.78:

Question 11:

If f : R → R is defined by f(x) = 6 – (x – 9)3, then f–1(x) = ___________.

ANSWER:

Given: A function f : R → R is defined by f(x) = 6 – (x – 9)3

f(x)=6-(x-9)3⇒y=6-(x-9)3⇒(x-9)3=6-yx-9=(6-y)13⇒x=9+(6-y)13Thus, f-1(x)=9+(6-x)13

Hence, if f : R → R is defined by f(x) = 6 – (x – 9)3, then f1(x) = 9+(6-x)13.

Page No 2.78:

Question 12:

Let A = {1, 2, 3, 4} and f : A → be given by f  = {(1, 4), (2, 3), (3, 2), (4, 1)}. Then f–1 = ___________.

ANSWER:

Given: A function f : A → be given by f  = {(1, 4), (2, 3), (3, 2), (4, 1)}

f={(1, 4), (2, 3), (3, 2), (4, 1)}⇒f-1={(4, 1), (3, 2), (2, 3), (1, 4)}Thus, f-1={(4, 1), (3, 2), (2, 3), (1, 4)}

Hence, f-1={(4, 1), (3, 2), (2, 3), (1, 4)}.

Page No 2.78:

Question 13:

If f : R → R be defined by f(x) = (2 – x5)1/5, then fof(x) = ___________.

ANSWER:

Given: f(x) = (2 – x5)1/5

fof(x)=f(f(x))         =f((2-x5)15)         =[2-((2-x5)15)5]15         =[2-(2-x5)15×5]15         =[2-2+x5]15         =[x5]15         =x5×15         =x


Hence, if f : R → R be defined by f(x) = (2 – x5)1/5, then fof(x) = x.

Page No 2.78:

Question 14:

Let A = {1, 2, 3, 4, 5, …, 10} and A → be an invertible function. Then, ∑10r=1(f−1of) (r)=___________.

ANSWER:

Given: A → is an invertible function, where A = {1, 2, 3, 4, 5, …, 10}

Since, f is invertibleTherefore, f-1of(x)=x       …(1)Now,∑10r=1(f-1of)(r)=f-1of(1)+f-1of(2)+f-1of(3)+….+f-1of(10)                   =1+2+3+….+10             (From (1))                   =10(10+1)2                       (∵ 1+2+3+…+n=n(n+1)2)                   =5(11)                   =55


Hence, ∑10r=1(f−1of) (r)= 55.

Page No 2.78:

Question 15:

Let A = {1, 2, 3, 4, 5, 6) and B = (2, 4, 6, 8, 10, 12). If f : A → B is given by f(x) = 2x, then f–1 as set of ordered pairs, is ___________.

ANSWER:

Given: A functionf : A → B defined as f(x) = 2x, where A = {1, 2, 3, 4, 5, 6} and B = {2, 4, 6, 8, 10, 12}

Since, f(x)=2xTherefore,f={(1, 2), (2, 4), (3, 6), (4, 8), (5, 10), (6, 12)}Hence,f-1={(2, 1), (4, 2), (6, 3), (8, 4), (10, 5), (12, 6)}


Hence, if f : A → B is given by f(x) = 2x, then f–1 as set of ordered pairs, is {(2, 1), (4, 2), (6, 3), (8, 4), (10, 5), (12, 6)}.

Page No 2.78:

Question 16:

Let  = {(0, –1), (–1, 3), (2, 3), (3, 5)} be a function from Z to Z defined by f(x) = ax + b. Then, (ab) = ___________.

ANSWER:

Given:  = {(0, –1), (–1, –3), (2, 3), (3, 5)} is a function from Z to Z defined by f(x) = ax + b


 = {(0, –1), (–1, –3), (2, 3), (3, 5)} defined by f(x) = ax + b

f(0)=-1⇒a(0)+b=-1⇒0+b=-1⇒b=-1          …(1)f(2)=3⇒a(2)+b=3⇒2a+b=3⇒2a-1=3             (From (1))⇒2a=3+1⇒2a=4⇒a=2               …(2)Thus,a=2 and b=-1


Hence, (ab) = ​(2, ​–1).


Disclaimer: The function f must be equal to  =  {(0, –1), (–1, –3), (2, 3), (3, 5)}.

Page No 2.78:

Question 17:

Let f : R → R and g : R → R be functions defined by f(x) = 5 – x2 and g(x) = 3x – 4. Then the value of fog (–1) is ___________.

ANSWER:

Given: f(x) = 5 – x2 and g(x) = 3x – 4


fog(-1)=f(g(-1))            =f(3(-1)-4)            =f(-3-4)            =f(-7)            =5-(-7)2            =5-49            =-44


Hence, the value of fog (–1) is ​–44.

 

Page No 2.78:

Question 18:

Let f be the greatest integer function defined as f(x) = [x] and g be the modules function defined as g(x) = |x|, then the value of gof(−54) is ___________.

ANSWER:

Given: f(x) = [x] and g(x) = |x|


gof(-54)=g(f(-54))              =g([-54])              =g(-2)              =|-2|              =2


Hence, the value of gof(−54) is ​2.
 

Page No 2.78:

Question 19:

If f(x) = cos [ex + cos [–ex, then f(π) = ___________.

ANSWER:

Given: f(x) = cos[e]x + cos[–e]x


f(x)=cos[e]x+cos[-e]x              =cos2x+cos(-3x)       (∵ e=2.718 approx)      =cos2x+cos3xf(π)=cos2π+cos3π           =1-1           =0


Hence, f(π) = ​0.

Page No 2.78:

Question 20:

Let A = {1, 2, 3} and B = {ab} be two sets. Then the number of constant functions from A to B is ___________.

ANSWER:

Given: Sets A = {1, 2, 3} and B = {ab}

Two constant functions can be formed from A to B.
i.e., one is f(x) = a and other is f(x) = b

Hence, the number of constant functions from A to B is 2.

Page No 2.78:

Question 21:

If f(x) = cos [π2x + cos [–π2x, then f(π2)=______________.

ANSWER:

Given: f(x) = cos [π2x + cos [–π2x


f(x)=cos[π2]x+cos[-π2]x              =cos9x+cos(-10x)       (∵ π2=9.85 approx)      =cos9x+cos10xf(π2)=cos9(π2)+cos10(π2)             =cos9π2+cos5π             =0-1             =-1


Hence, f(π2)= ​–1.

Page No 2.78:

Question 22:

The number of onto functions from A = {a, bc} to B = {1, 2, 3, 4} is __________.

ANSWER:

Given: A function from A = {a, bc} to B = {1, 2, 3, 4}

If a function from A to B is onto, then number of elements of A ≥ number of elements of B.

But here, number of elements of A < number of elements of B

Thus, no onto function exist from A = {a, bc} to B = {1, 2, 3, 4}.

Hence, the number of onto functions from A = {a, bc} to B = {1, 2, 3, 4} is 0.

Page No 2.78:

Question 23:

If f(0, ∞) → R is given by f(x) = log10 x, then f–1(x) = ___________.

ANSWER:

Given: f(x) = log10x

f(x)=log10xy=log10xx=10yThus, f-1(y)=10y.

Hence,  f1(x) = 10x.

Page No 2.78:

Question 24:

If R+ → R is defined as f(x) = logx, then f–1(x) = ______________.

ANSWER:

Given: f(x) = log3x

f(x)=log3xy=log3xx=3yThus, f-1(y)=3y.

Hence, f1(x) = 3x.

Page No 2.78:

Question 25:

If R → Rg : R → R are defined by f(x) = 5x – 3, g(x) = x2 + 3, then (gof–1) (3) = _______________.

ANSWER:

Given: f(x) = 5x – 3 and g(x) = x2 + 3

f(x)=5x-3⇒y=5x-3⇒5x=y+3⇒x=y+35Thus, f-1(y)=y+35.Now,gof-1(3)=g(f-1(3))             =g(3+35)             =g(65)             =(65)2+3             =3625+3             =36+7525             =11125

Hence, gof1(3) = 11125.

Page No 2.78:

Question 26:

If R → R is given by f(x) = 2x + |x|, then f(2x) + f(–x) + 4x = _______________.

ANSWER:

Given: f(x) = 2x + |x|

f(x)=2x+|x|f(2x)=2(2x)+|2x|⇒f(2x)=4x+2|x|         …(1)f(-x)=2(-x)+|-x|⇒f(-x)=-2x+|x|        …(2)Now,f(2x)+f(-x)+4x=4x+2|x|-2x+|x|+4x                           =6x+3|x|                           =3(2x+|x|)                           =3f(x)

Hence, f(2x) + f(–x) + 4x = 3f(x).

Page No 2.78:

Question 27:

If f(x)=1−x1+x, then fof(cos 2θ) = ______________.

ANSWER:

Given: f(x)=1−x1+x

fof(cos2θ)=f(f(cos2θ))                =f(1-cos2θ1+cos2θ)                =f(2sin2θ2cos2θ)                =f(tan2θ)                =1-tan2θ1+tan2θ                =cos2θ

Hence, fof(cos 2θ) = cos2θ.

Page No 2.78:

Question 28:

Let f(x)=xx−1andf(α)f(α+1)=f(αk), then k = ______________.

ANSWER:

Given: f(x)=xx−1andf(α)f(α+1)=f(αk)

f(a)f(a+1)=aa-1a+1a+1-1             =aa-1a+1a             =a2(a-1)(a+1)             =a2a2-1             …(1)                It is given that,f(a)f(a+1)=f(ak)⇒a2a2-1=akak-1⇒k=2

Hence, k = 2.

Page No 2.78:

Question 29:

If f (f(x)) = x + 1 for all x ∈ R and if f(0)=12, then f(1) = ____________.

ANSWER:

Given: f (f(x)) = x + 1 for all x ∈ R and f(0)=12

f(f(x))=x+1⇒f(f(0))=0+1⇒f(12)=1          (∵ f(0)=12)       ….(1)Now,f(f(12))=12+1⇒f(1)=1+22     (∵ f(12)=1)⇒f(1)=32

Hence, f(1) = 32.

Page No 2.78:

Question 30:

If f(x) = 3x + 10 and g(x) = x2 –1, then (fog)–1 is equal to ___________.

ANSWER:

Given: f(x) = 3x + 10 and g(x) = x2 –1

fog(x)=f(g(x))         =f(x2-1)         =3(x2-1)+10         =3x2-3+10         =3x2+7Thus, fog(x)=3x2+7⇒y=3x2+7⇒3x2=y-7⇒x2=y-73⇒x=±√y-73fog-1(x)=±√x-73

Hence, (fog)–1 is equal to ±√x-73.

Page No 2.78:

Question 31:

Let f = {(1, 2), (3, 5), (4, 1)) and g = {(2, 3), (5, 1), (1, 3)}. Then, gof = __________ and fog = __________.

ANSWER:

Given: f = {(1, 2), (3, 5), (4, 1)) and g = {(2, 3), (5, 1), (1, 3)}

fog(x)=f(g(x))fog(1)=f(g(1))         =f(3)         =5fog(2)=f(g(2))         =f(3)         =5fog(5)=f(g(5))         =f(1)         =2Hence, fog={(1, 5), (2, 5), (5, 2)}gof(x)=g(f(x))gof(1)=g(f(1))         =g(2)         =3gof(3)=g(f(3))         =g(5)         =1gof(4)=g(f(4))         =g(1)         =3Hence, gof={(1, 3), (3, 1), (4, 3)}.

Hence, gof = {(1, 3), (3, 1), (4, 3)} and fog = {(1, 5), (2, 5), (5, 2)}.

Page No 2.79:

Question 1:

Which one of the following graphs represents a function?

       

ANSWER:

In graph (b), 0 has more than one image, whereas every value of x in graph (a) has a unique image.
Thus, graph (a) represents a function.
So, the answer is (a).

Page No 2.79:

Question 2:

Which of the following graphs represents a one-one function?

 

ANSWER:

In the graph of (b), different elements on the x-axis have different images on the y-axis.But in (a), the graph cuts the x-axis at 3 points, which means that 3 points on the x-axis have the same image as 0 and hence, it is not one-one.

Page No 2.79:

Question 3:

If A = {1, 2, 3} and B = {ab}, write the total number of functions from A to B.

ANSWER:

Formula:
If set A has m elements and set B has n elements, then the number of functions from A to B is nm.
Given:
A = {1, 2, 3} and B = {ab}
n(A) = 3 and  n(B) = 2
∴ Number of functions from A to B = 23 = 8

Page No 2.79:

Question 4:

If A = {abc} and B = {−2, −1, 0, 1, 2}, write the total number of one-one functions from A to B.

ANSWER:

Let f:AB be a one-one function.
 Then, f(a) can take 5 values, f(b) can take 4 values and f(c) can take 3 values.

Then, the number of one-one functions = 5 × 4 × 3 = 60

Page No 2.79:

Question 5:

Write the total number of one-one functions from set A = {1, 2, 3, 4} to set B = {abc}.

ANSWER:

has 4 elements and B has 3 elements.
Also, one-one function is only possible from A to if n(A)≤n(B).
But, here n(A)>n(B).
So, the number of one-one functions from A to is 0.

Page No 2.79:

Question 6:

If f : R → R is defined by f(x) = x2, write f−1 (25).

ANSWER:

Let f-1(25)=x            … (1)⇒f(x)=25⇒x2=25⇒x2-25=0⇒(x-5)(x+5)=0⇒x=±5⇒f-1(25)={-5, 5}       [from (1)]

Page No 2.79:

Question 7:

If f : C → C is defined by f(x) = x2, write f−1 (−4). Here, C denotes the set of all complex numbers.

ANSWER:

Let f-1(-4)=x                  … (1)⇒f(x)=-4⇒x2=-4⇒x2+4=0⇒(x+2i)(x-2i)=0          [using the identity: a2+b2=(aib)(a+ib)]⇒x=±2i                           [as xC]⇒f-1(25)={-2i, 2i}         [from (1)]

Page No 2.80:

Question 8:

If f : R → R is given by f(x) = x3, write f−1 (1).

ANSWER:

Let f-1(1)= x           … (1)⇒f(x)= 1⇒x3= 1⇒x3-1= 0⇒(x-1)(x2+x+1)= 0        [using the identity:a3-b3=(ab)(a2+ab+b2)]⇒x=1          ( as xR) ⇒f-1(1)= {1}               [from (1)]

Page No 2.80:

Question 9:

Let C denote the set of all complex numbers. A function f : C → C is defined by f(x) = x3. Write f−1 (1).

ANSWER:

Let f-1(1)=x                   … (1)⇒f(x)=1⇒x3=1⇒x3-1=0⇒(x-1)(x2+x+1)=0        [Using identity: a3-b3=(ab)(a2+ab+b2)]⇒(x-1)(xω)(xω2)=0, where ω=1±i√32⇒x=1, ω or ω2                 (as xC)⇒f-1(1)={1, ωω2}             [from (1)]

Page No 2.80:

Question 10:

Let f  be a function from C (set of all complex numbers) to itself given by f(x) = x3. Write f−1 (−1).

ANSWER:

Let f-1(-1)=x                          … (1)⇒f(x)=-1⇒x3=-1⇒x3+1=0⇒(x+1)(x2-x+1)=0        [using the identity: a3+b3=(a+b)(a2-ab+b2)]⇒(x+1)(x+ω)(x+ω2)=0, where ω= 1±i√32    ⇒x=-1, –ω, –ω2        (as xC)⇒f-1(-1)={-1, –ω, –ω2}     [from (1)]

Page No 2.80:

Question 11:

If f : R → R be defined by f(x) = x4, write f−1 (1).

ANSWER:

Let f-1(1)=x             … (1)⇒f(x)=1⇒x4=1⇒x4-1=0⇒(x2-1)(x2+1)=0               [using identity: a2-b2=(ab)(a+b)]⇒(x-1)(x+1)(x2+1)=0         [using identity: a2-b2=(ab)(a+b)]⇒x=±1               [as xR]⇒f-1(1)={-1, 1}       [ from (1)]

Page No 2.80:

Question 12:

If f : C → C is defined by f(x) = x4, write f−1 (1).

ANSWER:

Let f-1(1)=x   … (1)⇒f(x)=1⇒x4=1⇒x4-1=0⇒(x2-1)(x2+1)=0                                                   [using identity: a2-b2=(ab)(a+b)]⇒(x-1)(x+1)(xi)(x+i)=0, where i=√-1           [using identity: a2-b2=(ab)(a+b)]⇒x=±1, ±i ⇒f-1(1)={-1, 1, i,-i}      [from (1)]

Page No 2.80:

Question 13:

If f : R → R is defined by f(x) = x2, find f−1 (−25).

ANSWER:

Let f-1(-25)=xf(x)=-25⇒x2=-25We cannot find x∈R, such that x2=-25       (as x2≥0 for all x∈R)So, f1(25)=ϕ

Page No 2.80:

Question 14:

If f : C → C is defined by f(x) = (x − 2)3, write f−1 (−1).

ANSWER:

Let f-1(-1)=x                              … (1)⇒f(x)=-1⇒(x-2)3=-1⇒x-2=-1 or –ω or –ω2         (as the roots of (-1)13are -1, -ω and -ω2, where ω=1±i√32)⇒x=-1+2 or 2-ω or 2-ω2=1,  2-ω, 2-ωf-1(-1)={1,  2-ω, 2-ω2}            [from (1)]

Page No 2.80:

Question 15:

If f : R → R is defined by f(x) = 10 x − 7, then write f−1 (x).

ANSWER:

Let f-1(x)=y                     … (1)⇒f(y)=x⇒10y-7=x⇒10y=x+7⇒y=x+710⇒f-1(x)=x+710             (From (1))

Page No 2.80:

Question 16:

Let f : (-π2, π2)→R be a function defined by f(x) = cos [x]. Write range (f).

ANSWER:

Domain =(-π2, π2)=(-1.57, 1.57)   (as π=227)So, cos [x]=cos (-2)=cos 2   ∀x∈(-1.57, 0)Also, cos 0=1 for x=0And cos [x]=cos 1  ∀x∈(0, 1.57)∴Range={1, cos 1, cos 2}

Page No 2.80:

Question 17:

If f : R → R defined by f(x) = 3x − 4 is invertible, then write f−1 (x).

ANSWER:

Let f-1(x)=y                 … (1)⇒f(y)=x⇒3y-4=x⇒3y=x+4⇒y=x+43⇒f-1(x)=x+43       [from (1)]

Page No 2.80:

Question 18:

If f : R → Rg : R → are given by f(x) = (x + 1)2 and g(x) = x2 + 1, then write the value of fog (−3).

ANSWER:

(fog)(-3)=f (g (-3))=f((-3)2+1)=f(10)=(10+1)2=121

Page No 2.80:

Question 19:

Let A = {x ∈ R : −4 ≤ x ≤ 4 and x ≠ 0} and f : A → R be defined by f(x)=|x|x. Write the range of f.

ANSWER:

f(x)=|x|xxx=±1 ∀xA, range of f={-1, 1}.

Page No 2.80:

Question 20:

Let f : [-π2, π2]→ A be defined by f(x) = sin x. If f is a bijection, write set A.

ANSWER:

f is a bijection,
co-domain of f = range of f
As -1≤sin x≤1,
-1≤y≤1

So, A = [-1, 1]

Page No 2.80:

Question 21:

Let f : R → R+ be defined by f(x) = axa > 0 and a ≠ 1. Write f−1 (x).

ANSWER:

Let f-1(x)=y                    … (1)⇒f(y)=xay=xy=loga xf-1(x)=log a x            [from (1)]

Page No 2.80:

Question 22:

Let f : R − {−1} → R − {1} be given by f(x)=xx+1. Write f-1 (x).

ANSWER:

Let f-1(x)=y                … (1)⇒f(y)=xyy+1=xy=xy+xyxy=xy(1-x)=xy=x1-xf-1(x)=x1-x            [from (1)]

Page No 2.80:

Question 23:

Let f : R-{-35}→R be a function defined as f(x)=2x5x+3.

Write f-1 : Range of fR-{-35}.

ANSWER:

Let f-1(x)=y              … (1)⇒f(y)=x⇒2y5y+3=x⇒2y=5xy+3x⇒2y-5xy=3xy(2-5x)=3xy=3x2-5xf-1(x)=3x2-5x   [from (1)]

Page No 2.80:

Question 24:

Let f : R → Rg : R → R be two functions defined by f(x) = x2 + x + 1 and g(x) = 1 − x2. Write fog (−2).

ANSWER:

(fog)(-2)=f (g (-2))=f(1-(-2)2)=f(-3)=(-3)2+(-3)+1=9-3+1=7

Page No 2.80:

Question 25:

Let f : R → R be defined as f(x)=2x-34. Write fof-1 (1).

ANSWER:

Let f-1(x)=y                …(1)⇒f(y)=x⇒2y-34=x⇒2y-3=4x⇒2y=4x+3⇒y=4x+32⇒f-1(x)=4x+32      [from (1)]⇒f-1(x)=4x+32∴(fof-1)(1)=f(4(1)+32)=f(72)=2(72)-34=7-34=44=1

Page No 2.80:

Question 26:

Let f be an invertible real function. Write

(f-1 of) (1) +(f-1 of) (2)+…+(f-1of) (100).

ANSWER:

Given that f  is an invertible real function.
f-1o f=I,where I is an identity function.So,(f-1o f)(1)+(f-1o f)(2)+…+(f-1o f)(100)=I(1)+I(2)+… +I(100)=1+2+…+100 (As I(x)=x, ∀xR)=100(100+1)2[Sum of first n natural numbers=n(n+1)2]=5050

Page No 2.80:

Question 27:

Let A = {1, 2, 3, 4} and B = {ab} be two sets. Write the total number of onto functions from A to B.

ANSWER:

Formula:

When two sets A and have m and elements respectively, then the number of onto functions from A to B is
{∑nr=1 (-1)r nCr rm, if mno, if m<n

Here, number of elements in A = 4 = m
Number of elements in B = 2 = n
So, m > n
Number of onto functions
=∑2r=1 (-1)r 2Cr r4=(-1)1 2C1 14+(-1)2 2C2 24=-2+16=14

Page No 2.80:

Question 28:

Write the domain of the real function f(x)=√x-[x].

ANSWER:

[x] is the greatest integral function.
So, 0≤x-[x]<1⇒√x-[x] exists for every xR.⇒Domain =R

Page No 2.80:

Question 29:

Write the domain of the real function f(x)=√[x]-x.

ANSWER:

[x] is the greatest integer function.
[x]≤x, ∀xR⇒[x]-x≤0,∀xR⇒√[x]-x does not exist for any xR.Domain =ϕ

Page No 2.80:

Question 30:

Write the domain of the real function f(x)=1√|x|-x.

ANSWER:

Case-1: When x>0|x|=x⇒1√|x|-x=1√xx=10=∞Case-2: When x<0|x|=-x⇒1√|x|-x=1√-xx=1√-2x (exists because when x<0, -2x>0)⇒f(x) is defined when x<0So, domain =(-∞,0)

Page No 2.80:

Question 31:

Write whether f : R → R, given by f(x)=x+√x2, is one-one, many-one, onto or into.

ANSWER:

f(x)=x+√x2=x±x=0 or 2xSo, each element x in the domain may contain 2 images.For example,f(0)=0+√02=0f(-1)=-1+√(-1)2=-1+√1=-1+1=0Here, the image of 0 and -1 is 0.
Hence, is may-one.

Page No 2.80:

Question 32:

If f(x) = x + 7 and g(x) = x − 7, x ∈ R, write fog (7).

ANSWER:

(fog)(7)=f ( g(7))=f(7-7)=f (0)=0+7=7

Page No 2.81:

Question 33:

What is the range of the function f(x)=|x-1|x-1?

ANSWER:

f(x)=|x-1|x-1=±(x-1)x-1=±1Range of f={-1, 1}

Page No 2.81:

Question 34:

If f : R → R be defined by f(x) = (3 − x3)1/3, then find fof (x).

ANSWER:

(fof) (x)=f (f (x))=f ((3-x3)13)=[3-((3-x3)13)3]13=[3-(3-x3)]13=(x3)13=x

Page No 2.81:

Question 35:

If f : R → R is defined by f(x) = 3x + 2, find f (f (x)).

ANSWER:

f(f (x))=f (3x+2)=3 (3x+2)+2=9x+6+2=9x+8

Page No 2.81:

Question 36:

Let A = {1, 2, 3}, B = {4, 5, 6, 7} and let f = {(1, 4), (2, 5), (3, 6)} be a function from A to B. State whether f is one-one or not.

ANSWER:

f = {(1, 4), (2, 5), (3, 6)}

Here, different elements of the domain have different images in the co-domain.
So, f is one-one.

Page No 2.81:

Question 37:

If f : {5, 6} → {2, 3} and g : {2, 3} → {5, 6} are given by f = {(5, 2), (6, 3)} and g = {(2, 5), (3, 6)}, then find fog.    [NCERT EXEMPLAR]

ANSWER:

We have,
f : {5, 6} → {2, 3} and g : {2, 3} → {5, 6} are given by f = {(5, 2), (6, 3)} and g = {(2, 5), (3, 6)}

As,
fog(2) = f(g(2)) = f(5) = 2,
fog(3) = f(g(3)) = f(6) = 3,

So,
fog : {2, 3} → {2, 3} is defined as
fog = {(2, 2), (3, 3)}

Page No 2.81:

Question 38:

Let f : R → R be the function defined by f(x) = 4x – 3 for all x ∈ R. Then write -1.                                                 [NCERT EXEMPLAR]

ANSWER:

We have,
f : R → R is the function defined by f(x) = 4x – 3 for all x ∈ R

Let f(x)=y. Then,y=4x-3⇒4x=y+3⇒x=y+34So, f-1(y)=y+34or, f-1(x)=x+34

Page No 2.81:

Question 39:

Which one the following relations on A = {1, 2, 3} is a function?
f = {(1, 3), (2, 3), (3, 2)}, g = {(1, 2), (1, 3), (3, 1)}                                                                                                         [NCERT EXEMPLAR]

ANSWER:

As, each element of the domain set has unique image in the relation f = {(1, 3), (2, 3), (3, 2)}

So, f is a function.

Also, the element 1 of the domain set has two images 2 and 3 of the range set in the relation g = {(1, 2), (1, 3), (3, 1)}

So, g is not a function.

Page No 2.81:

Question 40:

Write the domain of the real function f defined by f(x) = √25-x2.                                                                                 [NCERT EXEMPLAR]

ANSWER:

We have,f(x)=√25-x2The function is defined only when 25-x2≥0⇒x2-25≤0⇒(x+5)(x-5)≤0⇒x∈[-5, 5]So, the domain of the given function is [-5, 5].

Page No 2.81:

Question 41:

Let A = {abcd} and f : A → A be given by f = {(ab), (bd), (ca), (dc)}. Write -1.                                           [NCERT EXEMPLAR]

ANSWER:

We have,

A = {abcd} and f : A → A be given by f = {(ab), (bd), (ca), (dc)}

Since, the elements of a function when interchanged gives inverse function.

So, -1 = {(ba), (db), (ac), (cd)}

Page No 2.81:

Question 42:

Let fg : R → R be defined by f(x) = 2x + l and g(x) = x2 – 2 for all x ∈ R, respectively. Then, find gof.                 [NCERT EXEMPLAR]

ANSWER:

We have,

 fg : R → R are defined by f(x) = 2x + l and g(x) = x2 – 2 for all x ∈ R, respectively

Now,gof(x)=g(f(x))=g(2x+1)=(2x+1)2-2=4x2+4x+1-2=4x2+4x-1

Page No 2.81:

Question 43:

If the mapping f : {1, 3, 4} → {1, 2, 5} and g : {1, 2, 5} → {1, 3}, given by f = {(1, 2), (3, 5), (4, 1)} and g = {(2, 3), (5, 1), (1, 3)}, then write fog.                                                                                                                                                                           [NCERT EXEMPLAR]

ANSWER:

We have,

 f : {1, 3, 4} → {1, 2, 5} and g : {1, 2, 5} → {1, 3}, are given by f = {(1, 2), (3, 5), (4, 1)} and g = {(2, 3), (5, 1), (1, 3)}, respectively


As,

fog(2)=f(g(2))=f(3)=5,fog(5)=f(g(5))=f(1)=2,fog(1)=f(g(1))=f(3)=5,So,fog : {1,2,5}→{1,2,5} is given byfog={(2,5),(5,2),(1,5)}

Page No 2.81:

Question 44:

If a function g = {(1, 1), (2, 3), (3, 5), (4, 7)} is described by g(x) = αx+β, then find the values of α and β.                [NCERT EXEMPLAR]

ANSWER:

We have,

A function g = {(1, 1), (2, 3), (3, 5), (4, 7)} is described by g(x) = αx+β

As, g(1)=1 and g(2)=3So, α(1)+β=1⇒α+β=1         …..(i)and α(2)+β=3⇒2α+β=3       …..(ii)(ii)-(i), we get2αα=2⇒α=2Substituting α=2 in (i), we get2+β=1⇒β=-1

Page No 2.81:

Question 45:

If f(x) = 4 – (x – 7)3, then write f -1(x).                                                                                                                        [NCERT EXEMPLAR]

ANSWER:

We have,f(x)=4-(x-7)3Let y=4-(x-7)3⇒(x-7)3=4-yx-7=3√4-yx=7+3√4-yf-1(y)=7+3√4-y∴ f-1(x)=7+3√4-x

RD Sharma Solutions for Class 12 Maths Chapter 2: Download PDF

RD Sharma Solutions for Class 12 Maths Chapter 2–Functions

Download PDF: RD Sharma Solutions for Class 12 Maths Chapter 2–Functions PDF

Chapterwise RD Sharma Solutions for Class 12 Maths :

About RD Sharma

RD Sharma isn’t the kind of author you’d bump into at lit fests. But his bestselling books have helped many CBSE students lose their dread of maths. Sunday Times profiles the tutor turned internet star
He dreams of algorithms that would give most people nightmares. And, spends every waking hour thinking of ways to explain concepts like ‘series solution of linear differential equations’. Meet Dr Ravi Dutt Sharma — mathematics teacher and author of 25 reference books — whose name evokes as much awe as the subject he teaches. And though students have used his thick tomes for the last 31 years to ace the dreaded maths exam, it’s only recently that a spoof video turned the tutor into a YouTube star.

R D Sharma had a good laugh but said he shared little with his on-screen persona except for the love for maths. “I like to spend all my time thinking and writing about maths problems. I find it relaxing,” he says. When he is not writing books explaining mathematical concepts for classes 6 to 12 and engineering students, Sharma is busy dispensing his duty as vice-principal and head of department of science and humanities at Delhi government’s Guru Nanak Dev Institute of Technology.